bsol fall 2010 commercial law class notes i and 2

79
UCC Article 2 – Sales: Applies to Transactions in Goods Chapter 1 Basic Concepts Sec 2-105 Definitions (1) Goods means all things (including specially mfg’d goods) which are moveable at the time of identification to the contract, OTHER THAN money in which the price is to be paid, Investment Securities and Things in Action. “Goods” includes : Growing Crops; Unborn young of animals (but time limit must be existing and identified). ALSO: Specialty Goods, but exceptions apply, such as more stringent rules and Manufacturer can collect greater damages. Includes Tangible things – chattels Excludes Intangible things – Notes, CDs Section 2-104(1) Definition of Merchant (1) Person who deals in Goods of the kind or otherwise by his occupation holds himself out as having knowledge or skill particular to the Goods involved. NOTE : As soon as person is given position in your business, that person is presumed to have superior Knowledge to the customer Problem #1 Application of Article 2 to: a) Insurance policy – no – Intangible b) Real property – no: dirt is no; if house can be separated from realty, house w/b goods (mobile home) c) scale of building material Note: if you can separate it from house w/o causing destruction – you can take it – air conditioning unit on repossessed house – manufacturer (seller) can take it d) blood transfusion – more of a service- not really goods since only incidental to the predominate service e) false teeth unclear-would likely look to predominate factor f) Timber – yes, look to the ease of severability g) Computer program – yes (versus the service to go w/ it – no) h) Membership – no i) entire assets of clothing store – “Bulk Transfer/Sale” per UCC Article 6 AND also Goods per UCC2 j) Electricity – yes (goods) - 1 -

Upload: patrick-nolan

Post on 27-Apr-2015

1.160 views

Category:

Documents


0 download

TRANSCRIPT

Page 1: BSOL Fall 2010 Commercial Law Class Notes i and 2

UCC Article 2 – Sales: Applies to Transactions in Goods

Chapter 1 Basic Concepts

Sec 2-105 Definitions (1) Goods means all things (including specially mfg’d goods) which are moveable at the time of

identification to the contract, OTHER THAN money in which the price is to be paid, Investment Securities and Things in Action. “Goods” includes: Growing Crops; Unborn young of animals (but time limit must be existing and identified).

ALSO: Specialty Goods, but exceptions apply, such as more stringent rules and Manufacturer can collect greater damages.

Includes Tangible things – chattelsExcludes Intangible things – Notes, CDs

Section 2-104(1) Definition of Merchant(1) Person who deals in Goods of the kind or otherwise by his occupation holds himself out as having

knowledge or skill particular to the Goods involved.

NOTE: As soon as person is given position in your business, that person is presumed to have superior Knowledge to the customer

Problem #1 Application of Article 2 to: a) Insurance policy – no – Intangibleb) Real property – no: dirt is no; if house can be separated from realty, house w/b goods (mobile

home)c) scale of building material Note: if you can separate it from house w/o causing destruction –

you can take it – air conditioning unit on repossessed house – manufacturer (seller) can take itd) blood transfusion – more of a service- not really goods since only incidental to the predominate

servicee) false teeth unclear-would likely look to predominate factorf) Timber – yes, look to the ease of severabilityg) Computer program – yes (versus the service to go w/ it – no)h) Membership – noi) entire assets of clothing store – “Bulk Transfer/Sale” per UCC Article 6 AND also Goods per

UCC2j) Electricity – yes (goods)

MILAU ASSOC INC V NORTH AVE DEVELOP CORP (p. 3)FACTS: Massive burst in underground section of pipe connecting a sprinkler system to city water line on newly constructed building.

ISSUE: Issue was whether it was Services or Goods provided, given the service-oriented character of the transaction.

- 1 -

Page 2: BSOL Fall 2010 Commercial Law Class Notes i and 2

ANTHONY POOLS V SHEEHAN (p. 8)FACTS: Diving board at issue; diving board is “Good” as it is a mobile item.

Consumer Use – higher standards of responsibility.

Hybrid Transaction : Test is whether the predominant factor … the thrust, the purpose, reasonably stated is a Transaction of sale with labor incidentally involved or vice versa. Under the Predominate Purpose test, used by a majority of the Cts, a Hybrid transaction must be classified first as a Sale of Goods in order for the UCC to apply.

FINAL EXAM QUESTION:

Aunt B decides to have a yardsale. She sells a toaster oven that hasn’t been used in yrs – it explodes on new user – is Aunt B a Merchant – NO

Problem # 2 Private person sold his car to another person.UCC Article 2 apploes to all transactions of Goods –

Doesn’t always mean sale of Goods by Merchant- merely Sale of Goods

Yardsale? Do you have liability if having yardsale? No – Merchants are held to a greater standard of care

SIEMAN V ALDEN (p 19)FACTS: D wanted certain type of Rip saw. Couldn’t get saw he wanted from dealer, so went to another user and got different model from friend of seller.

Buyer got injured from saw – P was deemed not a Merchant w/r/t the sale of the saw (NOTE: D was a Merchant in his regular line of business) since didn’t sell saws as business.

NOTE: Person making an isolated sale of Goods is NOT a Merchant within the meaning of this section and thus NO Warranty of Merchantability would apply.

Sec 2-135 Implied Warranty; Fitness for Particular Purpose: requires (1) Seller knows of the particular purpose for which Goods are req’d(2) Buyer relies on Seller’s skill or Judgment in selecting the product

What constitutes a Dealer of Goods? Have a business location. A person making an isolated sale of goods in NOT a merchant…and thus no warranty of Merchantability would apply

NOTE: Defn of Merchant per Sec 2-104(1) refers NOT ONLY to those who deal in the Goods involved, but also those Merchants who deal in Practices of the kind involved in the Transaction.

Problem 3 Are the following Merchants?a) Person who quit job as teacher and opens Hat shop the next day.

Yes they are a Merchant and are deemed to know more than their customer even tho they are new to the business.

b) A farmer selling produce to a wholesaler…

- 2 -

Page 3: BSOL Fall 2010 Commercial Law Class Notes i and 2

Unclear - must determine whether “Casual Seller” or “Professional in business,” based upon facts and circumstances of the case.

Chapter 2 Contract Formation

A. STATUTE OF FRAUDS

ON EXAM:

Sec 2-201: Statute of Frauds

(1) Contract for Sale of Goods >$500 quantity NOT Enforceable UNLESS some writing sufficient to indicate a contract for sale has been made twix the Parties AND signed by Party against whom enforcement is sought (2) Twix Merchants, if w/in reasonable time a writing in confirmation of the contract and sufficient against the sender is received and the Party receiving it has reason to know of its contents, it satisfies the reqmnt of (1) unless written notice of objection w/in 10 days(3) A contract that does not meet (1) but is valid in other respects IS Enforceable

(a) if Goods are specially manufactured not suitable for others and Seller began work prior to Notice of repudiation is received

(b) if D admits that contract for sale exists in pleading, testimony or otherwise in Ct(c) w/r/t Goods where payment has been made and accepted

Problem #6 Situation where James Ross, president of James Ice cream called Robert Scott, president of Amud Ice Co and negotiated a purchase of 2 tons of ice at $256/ton. Mr Scott took the order over telephone, made notes on memo pad “2 tons ice-Ross Co,” initialed the memo pad and put memo on spindler marked “Orders.” Mr James Ross then wrote a letter to Amud Ice Co on 12/14/00 which confirmed their transaction describing it completely. On 1/17/01 Robert Scott phoned Mr Ross and denied existence of the contract detailed in Mr Ross’ previous letter. a. Does the Memo pad satisfy 2-201(1)?

Memo pad satisfies 2-201(1)

b. What legal Effect did Mr Ross’ 12/14/00 letter have? In order to have the letter satisft the S of F, it MUST state quantity in letter

c. Did Mr Scott’s denial of the terms contained in Mr Ross’ 12/22/00 letter avoid 2-201(2) ?Other Co’s denial of Order…2-201(2) 10 day response rule

STOPPED CLASS HERE

- 3 -

Page 4: BSOL Fall 2010 Commercial Law Class Notes i and 2

“Merchants Exception” to S of F 2-201(2) for Specially Manufactured Goods TWIX Merchants

Bazak International Corp v Mast Industries (p 31)HELD: Annotated PO forms signed by the Buyer, sent to Seller and retained w/out objection, fall w/in the Merchant’s Exception, satisfying the statutory requirement of a writing even w/out the Seller’s signature. In determining whether writings are confirmatory documents w/in 2-201(2) Neither explicit words of conformation Nor express references to the prior agreement are req’d AND the writings are sufficient so long as they afford a basis for believing that they reflect a real transaction twix the Parties.

KNOW FOR EXAMProblem # 7 City of Thebes ordered a water tank made in the shape of a golf ball on a tee [“Special Order Goods”] from Tanks America at a price of $30,000. City sent a down payment of $3,000. Tanks America was just about finished when a rep of the newly elected City administration called and told then the new administration considered the contract unenforceable. (i.e Repudiation of Order after down payment check received, and work on special order has begun)

a) Does the downpayment check satisfy 2-201(1)

Check for downpayment can satisfy reqmnts of 2-201(1); the quantity can be inferred b/c unique item

b) What legal argument can Seller make?Under 2-201(3)(a) Seller would argue work has begun on Special item prior to Repudiation AND that Goods are not available to sell to others in Sellers Ordinary Course of Business; Under 2-201(3)(c) Seller would argue that downpayment meets criteria stated.Ct have held that partial payment on a Non-Divisible item indicates sufficient contract for the sale of the whole item.

c) What if City has promised to sign a written contract, but had never gotten around to doing so. Could Promissory Estopple be used to circumvent 2-201?

Cts are inconsistent as to using Estopple argument for Seller; last ditch argument.

Problem #8 “Written JV agreement twix 2 Co; 1 to sell to the Other; no time period set”Is it service of goods?

Good Faith – as long as Merchants operate in Good Faith, they will have to pay.

This is NOT a Requirements contract because Parties have no agreement of exclusivity (no “all my requirements” or “all my output”).

Per 2-201(1) a Contract does NOT require that a quantity be stated in the contract; however, the contract will only be enforceable to the extent of the quantity stated in the Contract.

Problem # 9 Co decides to give up paperwork and go paperless; customers will place orders thru electronic media. Does this meet “written” requirement…

- 4 -

Page 5: BSOL Fall 2010 Commercial Law Class Notes i and 2

B. PAROLE EVIDENCE RULE

Sec 2-202 Parole Evidence Rule (pg. 42) Terms in a written memo of the Parties INTENDED by the Parties as a final expression of Agreement w/r/t such terms included therein may NOT be contradicted by evidence of any prior Agreement but MAY BE Explained:(a) by Course of Dealing – operate in past (how) or Course of performance (Contract controls unless

you can show you’ve done it that way before); and(b) by evidence of Consistent Addt’l Terms UNLESS Ct finds the writing to have been intended

ALSO as a complete and exclusive stmnt of the terms of the Agreement

Official Comment 3: If the addt’l terms are such that, if agreed upon, w/h/b included in the document in view of the Ct, then Evidence of their alleged making MUST be kept out.

(i.e. Material items MUST have been included in a Merged Contract)

Problem # 10 : Contract contains a Merger clause stating that all prior negotiations were merged into the written contract that contained terms of the agreement. a. Alleged precontract agreement to give Pres of buyer private flying lessons – may be considered a

consistent addtl termb. Alleged precontract agreement to allow buyer to return plane after 2 month if he didn’t like it –

would most likely be barred by the Parole Evidence rule. See Comment 3 to 2-202.

ON EXAM – MULTIPLE CHOICE:In this Order, Courts will look to determine if Merchant’s Exception applies:1. The language of the Contract itself2. Prior Dealings of the Parties3. Usage and Trade – Customs in the Industry

COLOMBIA v ROYSTER (p 43)HELD: What Columbia seeks to show is a practice of mutual adjustments so prevalent in the industry and in prior dealings twix the Parties that it formed a part of the agreement. Columbia’s evidence about Course of Dealing and Usage of Trade s/h/b admitted…

KEY: Quantity contract; Prior dealings; Course of usage When you sign output contract and market goes crazy – you have to keep on with contract

even though you’re losing a lot of money

C. OFFER AND ACCEPTANCE

Problem 11 Corp ordered fuses from Seller stating “reply by return mail.” Seller instead sent fuses which were found to be defective. Buyer had to procure the fuses elsewhere to Cover, and sued Seller for Breach of Warranty.a. When was the Contract formed?

Per 2-206(1)(b) at the moment of shipment Seller’s act of acceptance formed a contract. Altho order stated “reply by return mail” 2-206(1)(b) IS applicable because Buyer did not state “offer must be accepted in writing” or “reply by return mail ONLY.”

b. If instead Seller shipped different but similar fuses with note to “return if these don’t work” Seller has made a Counter Offer and NOT an acceptance. No Contract and No Breach. This is termed an Accommodation

- 5 -

Page 6: BSOL Fall 2010 Commercial Law Class Notes i and 2

Sec 2-206: Offer and Acceptance in Formation of Contract(1) UNLESS otherwise Unambiguously indicated by language

(a) Offer to make contract shall be construed as inviting Acceptance in any manner and any medium reasonable in the circumstances

(b) An Order to buy Goods for prompt shipment shall be construed as inviting Acceptance EITHER by Prompt promise to ship or prompt shipment of Conforming or Non Conforming Goods BUT shipment of Non Conforming Goods does NOT constitute Acceptance IF Seller seasonably notifies Buyer that shipment is offered only as an Accommodation to the Buyer.

Problem 12 : Car shopper asked for car that he wanted to buy to be held overnight so he could check with his wife. When he came back the next morning it was sold. Does he have a Cause of Action?

No cause of action and Sec 2-205 does NOT help, since no written signed agreement

Sec 2-205: Firm OffersAn Offer by a Merchant to Buy or Sell Goods in a Signed Writing, which by its terms gives assurances that it will be held open is NOT revocable, for lack of consideration during the time stated, or if no time is stated for a reasonable time, but in no event longer than 3 mths…

NOTE: Estopple c/b argued by an individual buyer since Seller’s in a stronger position than he is.

Problem # 13: Buyer of rugs had long and frequent dealings with Seller, and each sale was taken care of in the same way. Small print on the Seller’s order form provided that among other things, Seller disclaimed all Warranties, Express or Implied. On this order, the Accepted and paid-for carpeting proved to be non-conforming. Buyer sued Seller.

a. Was a contract formed?Contract was formed twix Buyer and Seller each time per 2-207(1)

b. Was Disclaimer part of that contract?Disclaimer of Warranties in was NOT part of sales contract because it was a Material alteration of the Contract and exercised under 2-207(2)(b), per Official Comment 4 to 2-207.

NOTE: The rest of the acceptance form is still an acceptance – FINDING A MATERIAL ALTERATION ONLY DROPS OUT THAT PARTICULAR TERM; REST OF CONTRACT STAYS.

Sec 2-207:Addt’l Terms in Acceptance or Confirmation1. A definite expression of Acceptance or a written confirmation which is sent within a reasonable

time operates as an Acceptance EVEN THO it states terms additional to or different form those offered UNLESS Acceptance is Expressly made Conditional on assent to the additional or different terms.

2. Addt’l Terms are to be construed as proposals for addition to the contract. [“Battle of the Forms”]Twix Merchants such terms become part of the contract UNLESS(a) offer Expressly Limits Acceptance to the terms of the offer(b) they Materially Alter it(c) Notification of Objection to them has already been given OR is given within a Reasonable

Time after Notice of them is received.

- 6 -

Page 7: BSOL Fall 2010 Commercial Law Class Notes i and 2

3. Conduct by both Parties which recognizes the existence of a contract IS Sufficient to establish a contract for sale although the writings of the Parties do not otherwise establish a contract.

In such case, the terms of the particular contract consist of those terms on which the writings of the Parties agree, together with any supplementary terms incorporated under the UCC

Official Comment 4: Examples of typical clauses which would ‘materially alter” the contract and so result in hardship if incorporated without Express awareness of the other Party are clauses:

1. Negating standard warranties as that of Merchantability or Fitness for a particular purpose2. Requiring 90 or 100% deliveries where Usage of trade allows greater quantity leeways3. Requiring that complaints be made in a time materially shorter than Customary or reasonable.

Official Comment 6: If no answer is received w/in a Reasonable time AFTER additional terms are proposed, it is both safe & commercially sound fair to assume that their inclusion has been assented to.

Problem # 14 : Seller demolished old bldgs and sent formal written offer and proposed to sell the bricks to Buyer on condition that Buyer would pick them up on site with delivery date of 6/15/01. Buyer accepted, enclosed a check but changed the delivery date to 7/20/01.

a. Will Buyer Breach the contract if he doesn’t pick up bricks by 6/15/01? Look to type of Goods involved if the proposed change clearly have a Major Dispute as to

Critical Term, it is a Key term to contract and change will result in NO Contract. In this situation the date change inconveniences a Party because Seller will be hindered

from reworking the site until the old bricks are hauled off.

DIAMOND FRUIT GROWERS v KRAK CORP (p 53)FACTS: Parties had been dealing with each other for 10 years and followed the same Course of Dealing during this time.

HELD: 2-207(1) converts a common law counteroffer into an Acceptance EVEN THO it states addt’l or different terms. NOTE THAT if expression of Acceptance Expressly Conditions Acceptance on Offeror’s assent to addt’l terms, Parties differing terms do NOT result in Contract UNLESS Offeror assents to the addt’l terms.

NOTE: Seller is more responsible for ambiguity because it inserts a term that requires assent and then doesn’t enforce it. If Seller truly does not want to be bound unless the Buyer assents to the terms, it can protect itself by NOT shipping until it obtains that assent.

Problem #15: Seller adds “Arbitration Clause” language in their acknowledgment of Buyers PO. Most Cts have held that this is a Material Alteration under 2-207 and is NOT a part of the Contract.

STOPPED CLASS HERE

- 7 -

Page 8: BSOL Fall 2010 Commercial Law Class Notes i and 2

DALE HORNING CO v FALCONER GLASS INDUSTRIES (p 60)DISC: Starting point is 2-715 which provides that Consequential Damages may be recovered for “any loss resulting from general or particular requirements and needs of which the seller at the time of contracting had reason to know and which could not reasonably be prevented by cover or otherwise.”

HELD: The lesson to be learned from this case is that merely inserting boilerplate provisions into standard forms is NOT the end all way to deal with the UCC. The only way to get a preferable term into a contract is to actually propose the term and reach a meeting of the minds on the issue.

Battle of the Forms: Per 2-207(2) where both Parties are Merchants such additional or different terms of each become part of the contract UNLESS they Materially alter the prior Agreement. An additional term is said to Materially alter a Contract if its incorporation into the K without the Express awareness of the other Party would result in surprise or Hardship.

Problem #16: On 4/20/01 Buyer sends PO with boilerplate language in bold letters stating “Buyer objects to any terms that differ from the PO.” On 5/3/01 Seller sends acknowledgment form with boilerplate language stating “this is not an acceptance unless Buyer assents to all changes made by this form.” Neither Party really read the other’s form. Seller shipped Goods on 5/6/01. a. Is there a Contract on 5/6/01?

Yes, per 2-207(3) shipment makes the contract. It will contain the terms from the Seller’s and Buyer’s contracts that agree and use the UCC to fill in the conflicting terms of the Contract.

b. Is there a Contract on 5/3/01? No, under 2-207(1) and (2)(a). Either Party could have backed out at this point in time.

LEONARD PEVAR CO v PRODUCTS CO (p 70) [Battle of the Forms}DISC: 2-207 recognizes that Buyer and Seller can enter into Contract one of three ways:

1. Parties agree orally and send confirmatory memo [2-207(1)]; 2. Parties without oral agreement may exchange writings that do NOT contain identical terms

but nevertheless constitute a seasonable acceptance [2-207(1)]; 3. Conduct of Parties may recognize existence of a Contract despite previous failure to

either orally or in writing agree, i.e. Performance (shipping the Goods) [2-207(3)].

- 8 -

Page 9: BSOL Fall 2010 Commercial Law Class Notes i and 2

Chapter 3 - Warranties

Two types of Warranties:

1) Title2) Quality

A. WARRANTY OF TITLE

Sec 2-312: Warranty of Title [NOTE: Title is Absolute: Either you Have Good Title or you Don’t](1) Subject to (2), there is, in a contract for Sale, a Warranty by Seller that

(a) Title conveyed shall be Good and its transfer Rightful(b) Goods shall be delivered free from any Security Interest or other Lien or Encumbrance…

(2) Warranty under (1) shall be modified ONLY by specific language or Circumstances that give the Buyer reason to know the Seller does NOT claim Title…(i.e. a Quit Claim Deed in Real Estate)

(3) UNLESS otherwise agreed, Seller who is a Merchant regularly dealing in Goods of the kind, Warrants that Goods shall be delivered Free of Claim…BUT a Buyer who furnished specifications to Seller MUST hold Seller harmless from Claims that arise out of compliance with the specifications.

NOTE: This is the only situation under the UCC where the Buyer is the Warrantor

Problem 17 : Stolen car sold to Pawn Car Co. (PCC). PCC was able to get title to car in Nevada, and sold to Joe. Joe resold to Ann, Police found car and took it. Ann sued Joe.

(a)(b) Joe argued that he had good title. Per 2-403 (1), Purchaser of Goods acquires all title that

Tr’or had… Since PCC had no title, neither Joe nor Ann had good title.(c)(d) Per 2-607(5)(a) where a Buyer is sued for Breach of Warranty for which his Seller is

answerable he may give his Seller written Notice of the suit and state that the Seller may come and defend and if Seller does not he is bound in any action against him by his Buyer based on determination of fact common to the 2 suits.

Problem 18 : Same facts as #17, except that police mistakenly impounded Joe’s car that was identical to the stolen car. Joe sent his legal bill to Motor Co, the seller of the car to Joe for payment(a) Motor Co is under no duty to pay it since the 3rd Party’s claim is spurious to Motor Co.

Problem 19 Determine if Warranty of Title is present in the following situatioins:(a) Sales Contract has clause “Product sold AS IS and Seller makes no Warranties, Express or

Implied as part of this sale.” No. ONLY applies to Warranty of Quality, NOT Warranty of Title.

(b)(c) Ted walks into bathroom at bus depot and is offered an expensive watch for $20.

No Warranty given in this type of situation.

- 9 -

Page 10: BSOL Fall 2010 Commercial Law Class Notes i and 2

B. WARRANTY OF QUALITY – Two Types: Express and Implied

1. Express Warranty of Quality – Seller does something Affirmative to create expectations about characteristic or performance of goods, advertisements. Verbal sales pitch of written contract, must have substance, relate to Goods.

Typically Seller will make oral or written reps in ads, the verbal sales pitch or written contract. These reps must have some substance to them (more than mere “puffing”) to be Express Warranty

If stmnt by Seller, however made, has ANY substance to it so that it might have played some part in Buyers decision to buy, burden is on the Seller to prove that Buyer did not rely. If Seller cannot meet this burden, Buyer has benefit of Express Warranty

Sec 2-313: Express Warranties by Affirmation (1) Express Warranties by Seller are created as follows:

(a) ANY affirmation of Fact or Promise made by Seller to Buyer which relates to Goods and becomes part of the basis of the bargain creates an Express Warranty that the Goods shall conform to the affirmation or promise.

(b) ANY description of Goods which is made part of the basis of the bargain creates an Express Warranty that the Goods shall conform to the Description

(c) Any sample or model which is made part of the basis of the bargain creates an Express Warranty that the whole shall conform to the model or sample

(2) …an affirmation merely of the value of the Goods or a statement purporting to be merely the Seller’s opinion or Commendation of the Goods does NOT create a Warranty

Official Comment 7 The precise Time when words are made or Samples are shown is NOT Material. The sole question is whether the Language or Samples are fairly to be regarded as part of the Contract. If language is used AFTER the closing of the deal (as when Buyer while taking delivery asks and receives additional assurances) the Warranty becomes a Modification and need NOT be supported by consideration if it is otherwise Reasonable and in order.

Problem 20: Are the following Express Warranties? [NOTE: Court will typically look to Buyer’s reliance on Seller, which is, in part, based upon the Buyer’s subjective knowledge.](a) Car Salesman told Buyer car was in “A-1” shape.

Since no stmnt as to operations of Car, mere puffing.(b) Farmer bought chickens that looked sickly; Seller said “With regular feed they would fatten up.”

Chickens later died in droves. Ct held this WAS Express Warranty, but since Oral, Jury must decide. If Stmnt was in writing would have been for Judge to decide if it was Express Warranty or not.

(c) Car Salesman told Buyer that “This is a great car. You’ll love it!” Most likely NOT an Express Warranty-more like “puffing.”

- 10 -

Page 11: BSOL Fall 2010 Commercial Law Class Notes i and 2

ON EXAMProblem 21: Customers bought wallpaper after being assured by salesman, after he found out that they had never put up wall paper before, that “The paper is easy to put up, you can use any paste and it dries immediately. It is the finest wallpaper.” They picked from a sample book. After contract signed, Salesman told them this type paper was used by a movie star. Paper tore easily, was hard to work with and was a different color than sample.

(a) Which Reps were an Express Warranty? KEY is that Customers’ lack of knowledge and reasonable Reliance on the Salesman for decision making. All but “It is the finest wallpaper”

(b) W/r/t/ “paper was used by a movie star” made after contract signedProbably NOT Express Warranty since made AFTER contract signed, UNLESS Buyer’s reliance thereon was obvious.

TEST TYPE QUESTION:

Ernest T Bass showed Barney samples of windows. As part of his sales pitch, he told B that the windows were so tough that B could not punch his fist thru them. As a demo, EB punched the window and it did not break. EB urged B to try; B tried, and true to EB’s word, B was unable to punch thru them. B ordered a set. Two days later EB delivers a set to B. Floyd comes by to look at the windows and B tells F how tough the windows are, convincing him to try and punch thru the window. When F tries his hand goes right thru the window severely injuring his haircutting hand…

TEST TYPE QUESTION:

Monica L buys a telephone where all telephone conversations are self erasing. M relied on salesman’s rep that the phone would prevent telephone conversations from being recorded. It didn’t work…

TEST TYPE QUESTION:

Bill Cosby goes into Hardware store to purchase a chainsaw to work with his ice sculpture art. He tells the salesman, Steve Erkul what he needs and instead of recommending the Sonbitch 2000 (made for ice sculpting) Steve recommends the Sonbitch 3000 instead. Bill Cosby purchases and while using it is injured.

NOTE: The Salesman is automatically deemed to have superior knowledge to the customer.

Problem 22: Joe buys car from Motor Co relying on seller’s claims regarding the car’s qualities. When car was delivered the Warranty book in the glove compartment had very limited Warranties.(a) Is Joe bound by the written Warranties?

Cts have held that Warranty limitation found in a later delivered book is ineffective to modify the terms Expessly or Impliedly agreed upon in the Offer and Acceptance process.

Problem 23: Paul bought a wig from Hair Co. the wig changed colors slightly from season to season, yet Paul did nothing about it. While reading the paper one day he noticed an ad for the wig that stated the wig did not change color. Paul check back and found that Hair Co. had run the same ad the week before he bought his wig, but he had not seen it. Paul sued Hair Co.

Since Paul cannot show Reliance on ad prior to his purchase…probably will not win.

- 11 -

Page 12: BSOL Fall 2010 Commercial Law Class Notes i and 2

2. Implied Warranty of Quality – Automatically part of contract UNLESS seller does something affirmative to get rid of them. Implied as matter of law.

a. Merchantability – the Goods must be Saleable AND conform to the normal expectations of the Parties. “fit for the ordinary purposes for which such goods are used”

KEY SECTION Sec 2-314: Implied Warranty: Merchantability: Usage of Trade

1. UNLESS Excluded or Modified [per 2-316(2), if Modification in writing MUST BE Conspicuous] a Warranty that Goods shall be Merchantable is Implied in a contract for their sale IF Seller is a Merchant w/r/t Goods of that kind. Serving for value of Food or Drink to be consumed either on the premises or elsewhere is a Sale

2. Goods to be Merchantable MUST be at least:a. pass w/out objection …under the contract descriptionb. for Fungible Goods are of fair average quality w/in the descriptionc. are Fit for the Ordinary Purposes for which Goods are usedd. run, w/in variations permitted by the agreement, of Even Kind, Quality and Quantity

within each Unit and among all Units involvede. are Adequately Contained, Packaged and Labeled as agreement may requiref. Conform to the promise or affirmations of Fact made on Container or Label

3. Unless excluded or modified other Implied Warranties may arise from Course of Dealing or Usage of Trade

TEST TYPE QUESTION on 2-314(2)(e):

Bottling Co delivers case of soda to store when the temperature is 105 degrees. After cases sit outside for alittle while, the storeowner brings them inside. A bottle explodes inside, injuring someone…

SHAFFER v VICTORIA STATION INC (p 83) [ON EXAM]FACTS: P ordered glass of wine; while sipping it the glass broke injuring P.

HELD: Sec 2-314(1) combined with 2-314(2)(e) addresses this situation. Drink c/n/b served without an adequate container. The drink sold included the wine sold AND the container BOTH of which must be fit for the ordinary purpose for which used.

Problem 24: (b) Joe, a policeman, sold a car to Mary, telling her it was a “good car.”

Since Joe is NOT a Merchant, NO Warrant of Merchantability under 2-313(2).

Problem 25: Nick was driving when a deer ran into his car; Nick was injured when he hit the edge of the door handle. Nick sued car mfgr for Breach of Merchantability.

See Larsen v General Motors 391 F 2d 495 (1968), where mfgr’s defense was that car was fit for its Ordinary Purpose and that it was misused.

- 12 -

Page 13: BSOL Fall 2010 Commercial Law Class Notes i and 2

Daniell v Ford Motor CO (p 89)FACTS: P became locked inside a car trunk for 9 days.

HELD: Implied Warranty of Merchantability requires that the product must be fit for the ordinary purposes for which Goods are used. The Implied Warranty of Merchantability does NOT require that the Buyer MUST prove reliance on the skill and judgment of the manufacturer.

b. Fitness for a Particular Purpose – where Buyer uses the Goods for something beyond their Ordinary purpose Warranty of Merchantability is not enough. But the Buyer MAY be able to sue for Breach of the Implied Warranty of Fitness for a Particular Purpose IF the Buyer can satisfy all the elements of 2-315.

Sec 2-315 Implied Warranty: Fitness for Particular PurposeWhere Seller AT THE TIME on contracting has Reason to Know ANY Particular Purpose for which the Goods are req’d AND that Buyer is relying on Seller’s skill or Judgment there is, UNLESS Excluded or Modified under 2-316 an Implied Warranty that Goods shall be fit for such purpose.

Problem 26: Chris built a recreation room and went to buy a heater for it; he saw an ad for A-1 Hotblast and went to Jones’ store. Jones knew the room as he had helped build it. Heater did not warm the room sufficiently.

KEY: What is asked by Buyer of the Merchant; did Merchant have duty to tell Buyer. If Buyer did not ask Merchant for assistance, it will be hard to prove Reliance of Buyer by Seller. Since the Heater functioned, 2-314 is N/A. Since Chris specified the Heater model, 2-315 is also most likely N/A.

ON EXAMProblem 27: Harold bought paint; he ran out half way thru painting his room. The addt’l paint was mixed at the same store. After finishing he noted new paint smelled and the color did not match the earlier paint.

Smelly problem breaches 2-314(2)(c) Non matching color breached 2-315

Problem 28: Donald ordered a martini; he bit the olive and cracked his tooth. Reasonable Expectations Test. Typically Cts will hold for D where “Natural” item caused

injury (pit in olive) for P where “Unnatural” item

STOPPED CLASS HERE

- 13 -

Page 14: BSOL Fall 2010 Commercial Law Class Notes i and 2

Webster v Blue Ship Tea Room Inc (p 94)FACTS: P had a fish bone lodged in her throat after eating fish Chowder. P sued for Breach of Implied Warranty…

HELD: Because P was from New England, she knew that Chowder could contain bones (she was held to a higher standard because of her knowledge). The occasional presence [of fish bones in Chowder] is to be anticipated and do not impair the Chowder’s Fitness of Merchantability.

NOTE: A Foreign/Unnatural object in a Taco would create liability to user.

ON EXAM Problem 29 : Carry bought hair dye which unbeknowst to her contained alcohol to which she was allergic causing damage to her scalp. Mfgr claimed that only .5% of population would have this allergic reaction.

(a) Allergic reactions are typically handled as strict liability. Merchantability focuses on fitness for Ordinary purposes. Issue is “what is out of the Ordinary?” NOTE: This is why we have such extensive labeling on products, or else a potential P

could show that Mfgr KNEW of the potential Risk and failed to warn user against it.

[Test type Question]3. Burden of Proof – In Warranty Suit the P has the Burden of proving:

a. Creation of Warrantyb. its Breachc. its Causal connection to P’s injury (Proximate cause)d. Fact and extent of the injury

ON EXAM Flippo v Mode O’Day Frock Shops of Hollywood (p 99)FACTS: P tried on slacks and was bit by a spider that was hiding in one of the pairs of slacks. As a result, P was hospitalized for 30 days. P sued on theory that clothes were unfit for Particular use for which they were intended.

HELD: Warranty of Implied Merchantability does not apply to this case. There is absolutely no evidence that the Goods were defective in any manner…there is no evidence that the mfgr or retailer had any control of the spider…

In cases involving sale and use of alleged Defective Product, there must be Substantial evidence of neglect, such Neglect being the proximate cause of the injuries sustained.

- 14 -

Page 15: BSOL Fall 2010 Commercial Law Class Notes i and 2

4. Warranty Disclaimers and Limitations (pg. 104)It is Unfair for a Seller to create an Express Warranty and then try to disclaim it; therefore UCC Section 2-316(1) was drafted in such a way as to make Disclaimer of Express Warranty virtually impossible.

ON EXAM: Proper way to avoid Liability for Express Warranty is NOT to make it in the first place. Express Warranties are created by Affirmative Seller conduct

Sec 2-316:Exclusion or Modification if Warranties (1) Words or…

(2) S:t (3) to exclude or Modify the Implied Warranty of Merchantability…the language must mention Merchantabilty AND in case of a writing must be Conspicuous (i.e Large Block letters in Red) and to Exclude or modify any Implied Warranty of Fitness, the Exclusion must be by a writing and conspicuous. For example: “There are no Warranties which extend beyond the description on the face hereof.”

(3) Notwithstanding (2) (a) Unless the circumstances dictate otherwise ALL Implied Warranties are excluded by

expressions like “as is” “with all faults” or other language which calls attention to the exclusion of Warranties and makes plain there is no Implied Warranty; and

(b) When the Buyer before entering into the contract examined the Goods or sample OR has refused to examine the Goods, there is NO Implied Warranty w/r/t defects which an examination ought to have revealed to him; and

(c) An Implied Warranty can also be Excluded or Modified by Course of Dealing or Course of Performance or Usage of Trade

(4) Remedies for Breach of Warranty can be limited in accordance with the provisions of this Article on liquidation or limitation of Damages AND on Contractual modification of remedy.

Problem 30: P went to buy a car and asked about gas mileage; salesman told he got twix 40-45 on hwy and 30-35 in city. Best she ever got was 27MPG. When she complained, dealer pointed to contract which said: (1) This is entire contract and there are no other matters agreed to by Parties not contained herein (2) There are no other Express Warranties except those contained herein (3) No salesperson has the authority to give Express Warranties other than those contained herein.

Did P rely on the statement made; did P have knowledge of transaction?Typically Cts would not take into account Parole Evidence..but would tend to look for equity in trying to recover for the consumer.

- 15 -

Page 16: BSOL Fall 2010 Commercial Law Class Notes i and 2

CATE (P) v. DOVER CORP (p 105)FACTS: Cate purchased 3 Lifts to elevate cars for maintenance.

DISC: Per 2-314(a) Unless Excluded or Modified [per 2-316(2) if Modification in writing MUST BE Conspicuous] a Warranty that Goods shall be Merchantable is Implied in a contract for their sale IF Seller is a Merchant w/r/t Goods of that kind. Term is Conspicuous if it is in Bold and Large writing – effect of this will be to Expressly Disclaim the Warranty.

A term or clause is Conspicuous when it is so written that a Reasonable Person against whom it is to operate OUGHT to have noticed it. NOT hidden among other attention getting language of the Warranty itself…Merely providing a Buyer a copy of documents containing an inconspicuous Disclaimer does NOT establish actual knowledge.

HELD: Merely providing a Buyer a copy of documents containing an Inconspicuous Disclaimer does NOT establish actual knowledge. Such a Disclaimer contained in text undistinguished in typeface, size or color within a form purporting to grant a Warranty is NOT Conspicuous and is Unenforceable UNLESS the Buyer has actual knowledge of the Disclaimer. Under 2-316 not much is actually required for an effective Disclaimer. To Disclaim the Implied Warranty the Seller need ONLY include word “Merchantability” in a Conspicuous fashion. Seller has Burden proving.knowledge of Disclaimer

Problem 31: Problems as follows:(a) Statement buried in fine print of used car purchase agreement “There are no Express or Implied

Warranties that are part of this sale”(1) Are Implied Warranties effectively Disclaimed?

No since buried in Contract.(2) How to redraft language?(3) What changes in the physical appearance of the clause?

To make effective, MUST be Conspicuous (Bold print, Different colored ink, Labeled as Warranty, mention “Merchantability”)

(4) Can car dealer argue “Usage of Trade?”(b) “AS IS” is written in soap in large letters on windshield of used car.

Implied Warranty, not Express Warranty(c) Salesman asks Buyer to examine the car; Buyer, who is in a hurry, declines. Effective Disclaimer?

An Inspection Disclaimer under 2-316 (3)(b) is valid ONLY where Seller has made demand for Buyer inspection.

NOTE that you look to the knowledge of the Buyer as part of whether Disclaimer is operative. (i.e. Auto mechanic buying a car can easily Disclaim)

BOWDOIN v. SHOWELL GROWERS (pg. 114)ISSUE: Implied Warranties of Fitness and Merchantablity. Disclaimer contained in instruction manual delivered with equipment, 2 weeks after purchase had been made.

HELD: Disclaimer MUST be Conspicuous to Purchaser PRIOR TO the sale. Court will generally hold that Disclaimer is ineffective because it did NOT form part of the Basis of the Bargain. Post-sale Disclaimer is ineffective – not a Basis of the Bargain

“Basis of the Bargain” rule protects Buyer from unexpected and coercive Disclaimers.

- 16 -

Page 17: BSOL Fall 2010 Commercial Law Class Notes i and 2

WILSON (P) v. DAVID FERGUSON (pg. 120)FACTS: D purchased yarn form D to knit into sweaters. After yarn was cut and sweaters were made the product was washed and it was discovered that the color of the yarn varied from piece to piece. And made the sweaters unsaleable. Sales contract had a 10 day time limitation on claims; D urged that this time period was unreasonable since color defect could not be discovered within this time period. D refused to pay for the yarn; P sued.

ISSUE: Yarn with color variability, not every defect is obvious – did they have a reasonable time to inspect goods? Latent defects are covered b/c they had no way of knowing

HELD: The time limitation clause of the Contract MUST give way to the general code rule that a Buyer has a Reasonable time to Notify the Seller of a Breach of Contract AFTER he discovers OR Should have discovered the defect. Contractual limitations upon remedies will be enforced UNLESS they are Unconscionable. Sec 2-316, Official Comment 1, provides that Warranty language prevails over Disclaimers if the 2 cannot be reconciled.

Problem #32: [BAR EXAM QUESTION]Consumer bought a snowmobile; contract he signed had warranty of Merchantability, stating in event of Breach “…remedy solely limited to repair or replacement of defective parts.” Contract Conspicuously stated “Seller is not responsible for any consequential damages.” Consumer took snowmobile back 3 times to get fixed; eventually engine blew up seriously injuring him.

(a) Per 2-719(3) and 2-715(2)(b) Consumer can recover for personal injury?NOTE: The more times that consumer takes the product to be repaired, without success, the more likely that Ct will hold Seller liable for costs.

Sec 2-715: Buyer’s Incidental and Consequential Damages (1) Incidental Damages resulting from Buyer’s Breach include…(2) Consequential Damages resulting from Buyer’s Breach include

(a) ANY loss resulting from general or particular requirements and needs, of which the Seller knew or had reason to know AND which could not reasonably be prevented by Cover or otherwise; AND

(b) Injury to person or Property proximately resulting from any Breach of Warranty.

See also: Pg. 125: tire blow-out cases where Personal injury occurred: Cts have held that tire mgfr could NOT avoid 2-719(3) even where its Disclaimer of Liability (Warranty) made a Conspicuous statement limiting the remedy for a blow out to repair or replacement, stating that this was NOT consistent with commercial and human realities”.

Pg. 126: tire blow-out which only totaled car: Ct found that the Warranty language limiting the Liability was “seriously lacking in clarity.”– Any remedy limitation MUST be Conspicuous EVEN THO 2-719 doesn’t explicitly say so.

Sec 2-718: Liquidation or Limitation of Damages; Deposits (1) Damages for Breach by either Party may be Liquidated in the Agreement but ONLY in an amount which is reasonable in the light of the anticipated or actual harm caused by the Breach…A term fixing Unreasonably large liquidated Damages is Void as a penalty.

Sec 2-719: Contractual Modification or Limitation of Remedy (3) Consequential Damages MAY be limited or Excluded UNLESS the Limitation or Exclusion is Unconscionable. Limitation of Consequential Damages for injury to the person in the case of Consumer Goods is prima facie Unconscionable BUT limitation of Damages where the loss is Commercial is not.

- 17 -

Page 18: BSOL Fall 2010 Commercial Law Class Notes i and 2

GODDARD v GENERAL MOTORS (p127 ) FACTS: New car had numerous repairs, some in excess of 20 days. P eventually bought another new car from same dealership, brought suit and was rewarded $5,000. Appellate Ct rev’d.

HELD: In accordance with 2-219(2), where a new car’s Express Warranty limits a Buyer’s remedy…but the new car is so riddled with defects that the limited remedy of repair fails its essential purpose, Buyer may institute an action to recover damages for Breach of Warranty and Consequential Damages under 2-714(3) and 2-715.

When a Seller is unable to fulfill its Warranted obligation to effectively repair or replace defects in Goods which are the subject matter of the sale, the Buyer is deprived of the benefits of the limited remedy and it therefore fails its essential purpose.

NOTE: Per Riley v Ford USCA 5th (Ala Case), which involved a Seller’s Express Warranty limiting the Buyer’s recourse to the repair or replacement of defective parts. Upon Seller’s inability to remedy at least 14 major and minor defects within a reasonable time period, Buyer brought action for Breach of Warranty against ford. Ct HELD that under the circumstances it was “unable to conclude that the jury ws unjustified in its implicit finding that the Warranty operated to deprive the Buyer of the “substantial value of the bargain.”

STOPPED CLASS HERE

- 18 -

Page 19: BSOL Fall 2010 Commercial Law Class Notes i and 2

5. Defenses in Warranty Actions

a. Notice – In Warranty action, the Buyer Loses ALL UCC rights if there is a failure to give Seller Notice of the Breach within a Reasonable period of time after the Breach should have been discovered. (1) Reason for this is to preserve for the Seller the Right to Inspect the Goods (2-515)

and Right to Cure (2-508)

Example: June 1, 1992: Professional team – 25 bats needed. Bats have to weight 48 oz. - #, price and description were on contract and Buyer had 10 days to reply if Goods were not conforming. Bat didn’t weigh enough. 15 unused by then – co. refused to replace or pay fine – team didn’t return within 10 days – no damages for team. “Cover” – get bats from another Co. and they cost more, so in SOME cases team may have to pay for difference.

Sec 2-607: Effect of Acceptance: Notice of Breach; Burden of Establishing Breach after Acceptance(1) Buyer must pay at the contract price for any Goods Accepted

(2) Acceptance of Goods by Buyer precludes Rejection of the Goods Accepted and if made with knowledge of a Non-Conformity CANNOT be Revoked b/c of it UNLESS Acceptance was on Reasonable assumption that the Non-Conformity w/b cured…

(3) Where a Tender has been Accepted(a) the Buyer MUST within a Reasonable time after he discovers OR should have discovered

ANY Breach, Notify the Seller of Breach or be barred from ANY Remedy

(4) The Burden is on the Buyer to establish any Breach w/r/t the Goods accepted…

Official Comment 4: The Time for Notification is to be determined by applying Commercial Standards to a Merchant Buyer. A “Reasonable Time” for Notification from a Retail Consumer is to be judged by different standards so that in his case it will be extended…not to deprive a Good Faith Consumer of his remedy. The content of the Notification need merely be sufficient to let the Seller know that the transaction is still troublesome and must be watched. There is no reason to require that the Notification …must include a clear statement of all the objections that will be relied upon by the Buyer…

Problem 33: [Bar Exam Question] Dave purchased shipment of apples from sample that Farmer showed him-he was told that actual apples w/b 20% smaller than those shown. Shipment was actually 33% smaller. Dave did not inspect them before he sent to be sold at market. Apples brought ½ of what Dave paid for them.

Should Dave have looked at apples? Apples didn’t reach conformity. Dave didn’t reply to Farmer about apples not being correct size. Have to tell Seller within 10 days of receiving Goods by Buyer. “Basis of Bargain” creates an Express Warranty.

NOTE: When Notice can be given but wasn’t..this is unfortunate for the person who had Opportunity to give Notice..Ct will typically rule against them.

NOTE: Different time period given based upon the type of Goodi.e. Bananas (which have spoilage) sitting on a dock have short period for Notice

than an order of Widgets (which have no spoilage). NOTE: Since perishable Goods involved, Buyer should have s/h/b inspected upon delivery.

- 19 -

Page 20: BSOL Fall 2010 Commercial Law Class Notes i and 2

Problem 34 Pg. 135 [Patterned after Eastern Airlines 532 F 2d 957] Airco ordered 40 planes form Daes Co; 20 to be delivered 5/8/01, balance 11/10/01. First batch of 20 came 9/11/01, but Airco did not complain. Balance came 1/12/02. On 1/30/02, Pres of Airco wrote Deas Co stating that poor delivery schedule cost them money, and, 3 mths later sued for damages caused by delayed delivery.

Airco waived its Rights by not complaining / providing Notice of late delivery of late planes. Airco s/h contacted Seller. Buyer should have said “Breach” in letter to manufacturer, since this does not mean that further performance must cease – may could work out pro-actively.

“Course of Dealing” doesn’t apply in this case – (i.e. “This is the way we’ve done it before”).1. only applies if contract is ambiguous2. see past course of dealings3. research industry policies on contract dealings

Problem 35 Pg. 136 Warranty extends to family or anyone who comes into contact with it – Sec 2-318 (A) – have to be injured by the Good.

Sec 2-607(5) – pg. 12- UCC – Notice must be given but court is lax on this. Should he give notice?

Sec 2-318 [Alternative A] Third Party Beneficiaries of Warranties Express or ImpliedA Seller’s Warranty whether Express or Implied extends to ANY natural person who is in the family or household of his Buyer or who is a guest in his home IF it is reasonable to expect that such person may use, consume or be affected by the Goods and who is injured in person by Breach of Warranty. A Seller may NOT exclude or limit the operation of this section.

Official Comment 3: This alternative Expressly includes as Beneficiaries within its provisions the Family, Household and Guests of the Purchaser. Beyond this the section is neutral and is NOT intended to enlarge or restrict the developing case law on whether the Seller’s Warranties, given to a Buyer who sells, extend to other persons in the distributive chain.

- 20 -

Page 21: BSOL Fall 2010 Commercial Law Class Notes i and 2

KEY ON EXAMb. Privity – MUST establish Contract twix the 2 Parties - can go “up ladder” to find liability (cause), There are 2 types of Privity:

(a) Vertical Privity – how far back up chain the Buyer can go (i.e. who to sue)

(b) Horizontal Privity – identifying to whom the Retail Seller is liable other than the immediate purchaser of the Goods

Problem 36 Girard Corp mfg’ed calculator. It bought paint used on it from Hamilton Paint Co. Girard Corp sold it to Leibnitz store, who in turn sold it to Syl who bought it for his wife, Joan. In turn their kids also used it. One day mailman used it…when he got home his dog licked his hand and dropped dead. Turned out that the paint was hazardous.

Warranty was to Syl, the one who purchased the Goods. Per Official Comment 3 to 2-318 it is extended to his family and others… Horizontal Privity

Mailman can sue “up the Chain”, thru Leibnitz store, thru Girard Corp up to Hamilton Paint Co…Vertical Privity

c. Note on Strict Products Liability – permits recovery by an injured Consumer in a suit against the Manufacturer as long as the Consumer can prove that the Manufacturer distributed into Commerce a product that contained a dangerous defect. There is no necessity of proving either Negligence OR Privity.

(1) If Consumer Goods, “Restatement of Torts 402A” comes into play. Strict Liability: Sec 402A Special Liability of Seller of Product for Physical Harm to User/Consumer

1. One who Sells any product in a Defective condition…is s:t liability for physical harm caused to the ultimate Consumer OR to his property IFa. Seller is engaged in Business of Selling the productb. It is expected to AND does reach the user without substantial change in

condition in which it is sold2. Above rule applies altho

a. Seller has exercised all possible care in preparation and sale of his product; &b. User has NOT bought the product from entered into contractl relation w/ Seller

(2) Cause of action based on Strict Liability is very similar to Implied Warranty of Merchantability. Differences twix Strict Liability and UCC Warranty are: [ON EXAM](a) Sec. 402A does NOT require Notice; UCC Cause of Action does(b) Sec 402A damages are limited to those for physical injury; Damages in UCC are not(c) Sec 402A has Statute of Limitations imposed by Tort law; UCC governed by 2-725

(significant time differences)(d) Sec 402A is NOT affected by Disclaimers or remedy limitations; UCC may also be

so limited(e) Privity is NOT an issue in Sec 402A; Privity may be an issue in UCC (f) 402A requires that product MUST contain a defect, UCC Warranty may be Breached

even if the product is NOT defective. i.e Sonbitch 2000 chain saw issue, where store salesman recommended the

wrong chain saw even tho it worked fine (but for wrong purpose in this instance)

- 21 -

Page 22: BSOL Fall 2010 Commercial Law Class Notes i and 2

EAST RIVER v TRANSAMERICA pg. 141Product Warranty ONLY applies to personal injury.

Looking to collect under Strict Product Liability – no personal injury

Problem 37 Pg. 151, Where axle on Car snapped, throwing car into pedestrian killing him. As pedestrian’s lawyer, under which theory should you sue?

Sue under BOTH Restmnt of Torts Sec 402A and UCC Sec 2-314? Have two separate Counts in the Complaint…Sue everybody

6. Magnuson-Moss Act (1975) – Applies to all Consumer products manufactured after 7/4/75 covered by a Written Warranty. Essentially deals with the Adequacy of information to consumers.(a) Stated purposes of the Statute are to improve the Adequacy of information available to

Consumers, to prevent Deception, to Improve adequacy of information available to Consumers to prevent Deception and to improve competition in the marketing of Consumer Products.

Problem 38: Does an Ambulance bought by a business fall within the Magnusin-Moss Act?Yes – Consumer is defined in the Act as the “Buyer…of any Consumer product” and Consumer product is defined as “any tangible personal property which…is NORMALLY used in personal, family or household purposes.”

Therefore even if product is NOT sold to an individual it can still be sold to a “Consumer”

Problem 39(a) Car Seller says “I refuse to Warranty my cars in any way.”

No Mag-Moss action since no Written Warranty; UCC 2-314 Warranty of Merchantability applies since it was NOT Disclaimed May be 2-313 Express Warranties if shown a model or sample.

(b) Car Seller says “I promise that car has engine that will last 5 yrs.” No Mag-Moss action since no Written Warranty; UCC 3-314 Warranty of Merchantability applies since it was NOT Disclaimed May be 2-313 Express Warranties if shown a model or sample.

(c) Car Seller writes “The only person to own this car was a little old lady…” Altho written, does not relate to “the nature of the material or workmanship” therefore

probably no Mag-Moss action. 2-313 Express Warranty applies

Problem 40 – pg 153 F bought car from D Motors. Car carried a full 3 month Warranty guarantying all parts except the tires. Warranty conspicuously stated that “This Warranty is in lieu of all other Warranties Express or Implied, particularly the Warranty of Merchantability.” 1 day after 3 months every part except the tires fails. Was Warranty Disclaimer effective?

NO – since Full Warranty given. Sec 2-314 – A reasonable period of time. The more expensive and more complex – longer period of time.

- 22 -

Page 23: BSOL Fall 2010 Commercial Law Class Notes i and 2

VENTURA v. FORD p 154Magnuson-Moss Warranty provides for 2 types of Written Warranties on Consumer Products:1. Full Warranty (per Sec 2304) –provides that Consumer MUST be given the election to receive a

refund or replacement without charge of a product that is Defective or malfunctions after reasonable attempts by Warrantor to fix

2. Limited Warranty (per Sec 2303) Warranties that do not meet 1, above, shall be Conspicuously designated a Limited Warranty, which protect Consumers by prohibiting Disclaimers of Implied Warranties (other than limiting in duration).

HELD: The Mag-Moss Act enhances Consumer’s position by allowing recovery under a Warranty without regard to Privity of contract twix Consumer and Warrantor by prohibiting the Disclaimer of Implied Warranties in a Written Warranty and by enlarging the remedies available to a Consumer for Breach of Warranty including the award of Attorney’s Fees.

Warrantor includes any Supplier or other person who gives or offers to give a Written Warranty or who is obligated under an Implied Warranty.

KEY: Mag-Moss Act– includes attorney fees into collection

STOPPED CLASS HERE

- 23 -

Page 24: BSOL Fall 2010 Commercial Law Class Notes i and 2

Summary of Class so Far…Potential Midterm Questions

Pg. 1 Problem 1Pg. 18 Merchants – saw question, merchants have higher level of responsibility, liability or not?Pg. 20 Consumer ProductsPg. 23 Definition Sec 2-201, 2-205Pg. 24 Are the following merchants?No scope of 2A and International SalesPg. 29 Statute of Frauds – know part from answer sheetPg. 42 Problem 10Pg. 49 Problem 11 – offer and acceptancePg. 77 Warranties, Problems 17 &19 also info after problemPg. 79 & 83 Express and Implied, express warranty or puffingPg. 83 MerchantabilityPg. 84 fish chowder, wine glass casePg. 92 fitness for a particular purpose, chainsaw for ice sculpting – not ordinary purpose Sec 2-315

40 multiple choice10 yes/nolots of warranty questionspg. 136 Privity, problem 36pg. 139 Sec 402A someone is injuredMagnason-Moss Act – get attorney’s fees

20 will be on Warranties – Merchantability, Disclaimers, Car cases - are they Goods or not?

- 24 -

Page 25: BSOL Fall 2010 Commercial Law Class Notes i and 2

CHAPTER 4 – TERMS OF THE CONTRACT

A. Filling in The Gaps - UCC is there to fill in gaps or problems with contracts.

Sec 2-305 Open Price Terms(1) Parties can conclude a Contract for sale EVEN THO the price is NOT settled. In such case the

price is a Reasonable price at the time of Delivery if:(a) Nothing is said as to Price; or(b) The Price is left to be agreed by the Parties and they fail to agree; or(c) The Price is to be fixed in terms of some Agreed Market or other standard as set or recorded

by a 3rd person or Agency and it is not so set or recorded.

(2) A Price to be set by Seller or Buyer is a price for him to fix in Good Faith

(3) When a price left to be fixed otherwise than by Agreement of the Parties fails to be fixed thru fault of one Party, the other Party may AT HIS OPTION threat the contract as cancelled OR himself fix a reasonable price.

(4) Where Parties intend NOT to be bound UNLESS the price be fixed or agreed AND it is not fixed or agreed, there is no contract. Buyer must return any Goods received or pay their Reasonable value at time of delivery and Seller must return any portion of the price paid in account.

Sec 2-311 Options and Cooperation Respecting Performance(1) An Agreement for sale which is otherwise sufficiently definite to be a contract is NOT made

invalid by the fact that it leaves particulars of performance to be specified by one of the Parties. Any such specification must be made in Good Faith and within limits set by Commercial reasonableness…

Problem. 42: Sec 2-305, 2-311 Ed, who has ordered for years [KEY is “Past Dealings” twix Seller and Buyer] from Watson Oil C, ordered 100 cases of oil from Watson Oil Co, some of type A and some of type B, saying he would let Oil Co know exact mix one week before delivery. Watson Oil Co set price for type B at $30 per case, but since price was wildly fluctuating on type A, price would have to be set at time of delivery. Parties signed written contract so indicating. Week before scheduled delivery, Oil Co calls Ed who says “April Fool-I’m not taking any oil.”(1) What is solution?

Watson Oil will be able to designate the mix of type A and B quantity of oil in the sale, based upon what Ed had typically ordered in the past [ Past Dealings]

IMPT under 2-311, Seller Must Act in “Good Faith”

(2) If Buyer had merely said “April Fools” what is solution? Buyer has to give a more definite answer or cancellation – Notice.

“April’s Fool” doesn’t work; Say – “We don’t need oil”Seller should send letter to Buyer telling it what Buyer intends to do with respect o shipping the order.

ALTERNATIVE: Would be for Seller to Mitigate Damages – sell to someone else and get difference from person that wouldn’t buy (person in breach)

- 25 -

Page 26: BSOL Fall 2010 Commercial Law Class Notes i and 2

LANDRUM v DAVENPORT p 178FACTS: P wanted to purchase 3 1977 Indy Pace cars Corvettes. He put $100 down, but Car Co left price blank, but stated sticker would be twix $14 -16K. When car arrived there was a huge demand for car-Car Co required $22K for it. P paid under protest and sued.

HELD: If both Parties agreed upon a price the failure to insert that agreed price in the written contract did NOT invalidate the agreement under the circumstances presented here. Even if price was not agreed upon, Agreement may still be binding if both Parties intended to be bound and there is reasonably certain basis for giving appropriate remedy.

There was no Novation here, as the old obligation was not replaced with a new obligation. Nor was there a Waiver since there was no voluntary relinquishment of a known right. There was also no Estopple, since it was not established conclusively that one Party act caused the other to act that would operate to his detriment if the other Party is allowed to refuse to act. Intention to ratify or Waive CANNOT be inferred from acts where, under the exigencies of the case, the party had no satisfactory alternative.

ON EXAM: Defn of NOVATION: “The voluntary replacement of an Old Obligation with a New Obligation.”

Sec 1-207 Performance or Acceptance Under Reservation of Rights(1) A Party who with explicit reservation of rights performs or promises performance or assents to

performance in a manner demanded or offered by the other Party does NOT thereby prejudice the rights reserved. Such words as “without prejudice” or “under protest” or the like are sufficient.

(2) The above does NOT apply to an accord and satisfaction.

Official Comment 1: All of the phrases completely reserve ALL rights within the meaning of this Section

Example:$800 owed. Check remitted as follows: “$600 paid in full.” In order to reserve rights, scratch out the words “paid in full,” then write “reserve rights” & you can cash the $600 now preserve rights on the remaining balance of $200 later

B. Unconscionability Pg. 183 – term is NOT defined in UCC. Prof Leff divided into 2 Types1. Procedural Unconscionability - Unfair conduct in the Formation of the contract2. Substantive Unconscionability – Unfairness in the terms of the resulting bargain

KEY DEFN: Unconscionability has generally recognized to include an absence of meaningful choice on the part of one of the Parties together with contract terms which are unreasonably favorable to the other Party. Whether a meaningful choice is present in a particular case can only be determined by consideration of all the circumstances surrounding the transaction. In many cases the meaningfulness of the choice is negated by a gross inequality of bargaining Power. Did each Party, considering his obvious education or lack of it, have a reasonable opportunity to understand the terms of the contract or were important terms hidden in a maze of fine print and minimized by deceptive sales practices?

i.e. Rent to own stores would try to get security interest in ALL Goods that they have provided to you, even tho the earliest ones are mostly paid for; upon any default they repossess all of the Goods that have been bought, not just the ones that are late.

Identification of goods Pg. 185Seller bears loss if still in possession

- 26 -

Page 27: BSOL Fall 2010 Commercial Law Class Notes i and 2

Sec 2-302 Unconscionable Contract or Clause(1) If the Ct as a matter of law finds the contract…to have been Unconscionable at the time it was

made, the Ct may refuse to enforce the contract, or it may enforce the remainder of the contract without the Unconscionable clause, or it may so limit the application of any Unconscionable clause…

Problem 43 Buyer told salesman he knew nothing about sailboats [KEY: Therefore Buyer would rely on Seller, as Buyer had unequal knowledge to Buyer] and wind up signing a purchase contract for a model costing $3150. On way home he passed another store that advertised the same boat at $1100. Subsequent investigation showed that other stores charged no more than $1250 for the same boat. Buyer had done no comparative shopping. Does 2-302 permit Buyer to void the sale?

There is Procedural Unconscionability in the Merchant’s Bad Faith appropriation of Buyer’s money when the Buyer’s lack of experience is obvious.

C. Identification of the Goods – UCC provisions draw distinctions based upon whether Goods have been Identified as the specific Goods to which the contract relates.

Sec 2-105 Defns: Goods(4). Undivided share in an identified Bulk of fungible Goods is sufficiently ID’ed to be sold altho

quantity of the bulk is not determined….

Sec 2-501 Insurable Interest in Goods; Manner of Identification of Goods1. Buyer obtains special property and Insurable Interest in Goods by identification of existing

Goods to which the contract relates even tho the Goods so identified are non-conforming and he has the option to return or reject them. Such ID can be made at any time and in any manner explicitly agreed to by the Parties. In the absence of explicit agreement, identification occurs:a. When contract is made if for sale of Goods already exist and ID’ed b. If Contract for sale of future Goods other than (c), when Goods are shipped, marked or

otherwise designated by Seller as Goods to which the contract refersc. When the

(i) Crops are planted or otherwise become growing crops OR(ii) Young conceived (if contract for sale of unborn young to be born within 12 mths after

contracting) OR(iii) For sale of crops to be harvested within 12 months or the next normal harvest season after

contracting, whichever is longer

Problem #44 When does ID occur?a) Seller is fisherman who contracts to sell entire season’s catch

ID occurs when fish caught - all means all – the fish does itb) Circus agrees to sell calf of its elephant; contract made when she was 2 mths pregnant.

Elephant gestation period is 20 months – statute doesn’t apply. ID would be upon birth.c) Farmer agreed to sell Co part of grain that he had stored in a silo, which was mixed with other

farmers’ grain (i.e. Fungible Good)Defn of Fungible Goods – defined as “where any unit is the equilavent of any other unit)

ID occurs upon contracting as long as quality of all grain is the sameIssue is how common are the goods

d) Co to sell 5000 widgets; it has 2M widgets in its warehouse, all alike.

- 27 -

Page 28: BSOL Fall 2010 Commercial Law Class Notes i and 2

D. Risk of Loss: No Breach – absent Contrary Agreement1. General Rules:

a. Where Seller is a Merchant, Risk of Loss passes to Buyer on Buyer’s actual Receipt of Goods

b. Where Seller is NOT Merchant, Risk of Loss passes to Buyer when Seller Tenders delivery.

Sec 2-503 Manner of Seller’s Tender of Delivery1. Tender of delivery requires that Seller put and hold conforming Goods at the Buyer’s disposition

and give Buyer any Notification reasonably necessary to enable him to take delivery…a. Tender MUST be at a reasonable hour and if it is of Goods they MUST be kept available for a

period reasonably necessary to enable the Buyer to take possession…

Sec 2-509 Risk of Loss in Absence of Breach1. Where contract requires or authorizes Seller to ship Goods by carrier

a. If it does NOT require him to deliver them at a particular destination, the Risk of Loss passes to Buyer when Goods are delivered to the Carrier…BUT

b. If it DOES require him to deliver them at a particular destination, and the Goods are there Duly Tendered while in the possession of the Carrier, the Risk of Loss passes to Buyer when Goods are there Duly Tendered as to enable Buyer to take delivery.

2. Where Goods are held by a Bailee to be delivered without being moved, Risk of Loss passes to Buyer a. On his receipt of a Negotiable Document of Title covering the Goodsb. On Acknowledgement by the Bailee of the Buyer’s rights to possession of the Goods; orc. After his receipt of a Non-Negotiable Document of Title…

3. In any case not within (1) or (2) Risk of Loss passes to Buyer on his receipt of Goods IF Seller is Merchant; otherwise Risk of Loss passes to Buyer on Tender of Delivery.

4. Provisions are s:t contrary agreement of the Parties…

Official Comment 2 The provisions of subsection (1) apply where...To transfer the Risk it is enough that a proper shipment and proper ID come to apply to the same Goods altho, aside from special agreement, the Risk will NOT pass retroactively to the time of shipment in such a case.

Sec 2-510 Effect of Breach on Risk of Loss(1) Where a Tender or Delivery of Goods so fails to conform to the contract as to give a Right of

rejection, the Risk of their Loss remains on the Seller until Cure or Acceptance

(2) Where Buyer rightfully revokes Acceptance, he may to the extent of any deficiency in his effective insurance coverage, treat the Risk of Loss as having rested on the Seller from the beginning.

(3) Where the Buyer as to Conforming Goods already ID’ed to the contract for sale repudiates or is otherwise in Breach before Risk of their Loss has passed to him, the Seller may to the extent of any deficiency in his effective insurance coverage treat the Risk of Loss as resting on the Buyer for a commercially reasonable time.

- 28 -

Page 29: BSOL Fall 2010 Commercial Law Class Notes i and 2

Problem 45 p187Joe orders car from Honest John Car Dealer. Salesman calls Joe to tell him his car has arrived. Joe is too busy to pick up and will get tomorrow. Overnight the car is stolen. Who bears Risk of Loss?

Since Seller is Merchant, it bears loss while Goods are still in its possession; upon receipt of the Goods by Buyer, Risk of Loss transfers to him.

Problem 46 p187Janice decided to have a Garage sale. Her neighbor agreed to buy her piano and was to have it moved the next day. That night Janice’s house burned and piano was destroyed.

Where Tender was to pass the next day, Since Seller is NOT a Merchant, Risk of Loss passes to Buyer when Seller Tenders delivery

Where Tender was to pass in 6 mths, Ct would be more likely to say that Risk of Loss passes to Buyer when commercially reasonable time, per 2-709(1)(a) for performance passed.

Jason’s Food v. Peter Eckrich & Sons HELD: 2-509(2) separates Title from Risk of Loss. Title to the ribs passed to Eckrich when the warehouse made the transfer on its books from Jason’s account to Eckrich’s, but the Risk of Loss did NOT pass until the transfer was “acknowledged.”

2. Delivery Terms (KNOW FOR TEST QUESTION)a. Shipment Contract – Seller need only get the Goods to the Carrier and then the Buyer will

take the Risk of Loss – indicated by “CIF” of “C&F”b. Destination Contract – where Goods must be delivered by the Carrier before Risk of Loss

passes to the Buyer.

Example: If Seller is in New York and Buyer is in Chicago, “$2,000 FOB New York” means that the Risk of Loss passes on delivery to the Carrier AND that the price quote includes ONLY delivery to the Carrier (Buyer must pay freight and insure Goods while in transit).

Sec 2-319 FOB and FAS Terms1. UNLESS otherwise agreed, term FOB at a named place is a Delivery Term under which

[NOTE: The named place is where Risk of Loss Passes]a. when FOB the place of Shipment, Seller MUST at that place ship the Goods…and bear the

Expense and Risk of putting them onto the possession of the Carrier;b. when FOB the place of Destination, Seller MUST at his own Expense and Risk transport the

Goods to that place and there tender delivery …c. when under (a) and (b) the term is also FOB vessel, car or other vehicle, Seller must also at his

own expense and Risk load the Goods on board…2. Unless…the term FAS (free along side) vessel at a named port is a delivery term under which the

Seller must:a. at his own expense and Risk deliver the Goods alongside the vessel…on a dock designated and

provided by Buyer; andb. obtain a receipt for Goods in exchange for which Carrier is under a duty to issue a bill of lading.

- 29 -

Page 30: BSOL Fall 2010 Commercial Law Class Notes i and 2

Problem 47 Seller in New York contracted to buy clothing from Seller in Savanah, terms “$1,800 FAS S.S. Seaworthy, New York City.” Seller delivered the boxes of clothing to the dock and obtained a bill of lading. Before boxes could be loaded, dock collapsed. Must the Buyer pay the $1,800 anyway?

Yes – Buyer must pay since “FAS S.S. Seaworthy” – Seller, per 2-319(2), has done all it needs to pass the Risk.

Problem 48 Seller in Detroit contracted to sell 50 cars to Buyer in B’ham. After carrier has received the cars, but before they are loaded on the train for shipment, lightening strikes them, destroying them. a. If shipped “FOB Detroit” who had Risk of Loss?

Per 2-319(a)(1) Buyer has Risk of Lossb. If shipped “FOB railroad cars, Detroit” who has Risk of Loss?

Per 2-319(a)(3) Seller has Risk of Loss until Goods loaded on Carrier.c. If shipped “CIF Birmingham” who has Risk of Loss?

Since B’ham paid Insurance (Cost Ins Freight) Risk of Loss passed to Buyer

Problem 49 Pineapple Inc. loaded 5 boxcars of pineapples on independent carrier when he received notice that another customer agreed to buy a boxcar load to Grocery King “FOB Seller’s processing plant.” Pineapple agreed to diverted a boxcar load to Grocery King, but before it could the entire 5 boxcars were destroyed by hurricane. Who bears the Risk of Loss?

Per 2-501(1) since Goods were NOT ID’ed Seller retains the Risk of Loss. This is NOT the sale of an undivided share, since goods were segregated into separate boxcars.

Sec 2-504 Shipment by SellerWhere the Seller is req’d to send the Goods to the Buyer and the contract does NOT require him to deliver them at a particular destination, then UNLESS otherwise agreed, he MUST:

a. put the Goods in the possession of…Carrier AND make such a contract for their transportation as may be Reasonable having regard to the Nature of the Goods…;and

b. obtain and promptly deliver…any documentation…; andc. promptly notify the Buyer of the shipment.

Failure to Notify under (c) or make proper contract under (a)…is grounds for rejection ONLY if Material delay or loss ensues.

Cook Speciality Co v. Schrlock (p 197)FACTS: P entered into sales contract with D to buy a machine for $28K which was lost in shipment. Terms of contract called for FOB D’s warehouse. Carrier’s ins co paid only $5K.

HELD: Clear implication is that the reasonableness of a shipper’s conduct under 2-504 is determined with regard to the mode of transportation selected. It would be Unreasonable, for example, to send perishable without refrigeration. Contract in this case was FOB Seller’s warehouse. P clearly bears the Risk of Loss in transit.

Rheinberg –Kellerer (P) v. Vineyard Wine Co (D) (p 204)HELD: P’s failure to Notify D of the shipment until after the sailing of the ship and the ensuing loss, was NOT “prompt Notice” within the meaning of 2-504, and therefore, the Risk of Loss did not pass to D upon the delivery of the wine to the Carrier pursuant to the provisions of 2-509(1)(a). Since the D was never Notified directly or by the forwarding of shipping documents within the time in which its interest could have been protected by insurance or otherwise, D was entitled to reject the shipment pursuant to the term of 2-504(c)

- 30 -

Page 31: BSOL Fall 2010 Commercial Law Class Notes i and 2

Problem 50 The Univ of China ordered video eqpt to be shipped from Allied Tech in California. Parties had not gotten around to signing the contract until the Goods were already on board the plane crossing the Pacific ocean. Does the Buyer have the Risk of Loss only from the moment the contract is signed OR from delivery of the Goods to the air carrier?

CHAPTER 5 – PERFORMANCE OF THE CONTRACT

EXAM QUESTION:Sec 2-301 states that

Seller’s basic obligation is to Transfer and DeliverBuyer’s basic obligation is to pay in accordance with the Contract.

Thus the Seller must Tender (offer to deliver) conforming Goods and the Buyer must pay for them.

A. Installment Sales – Substantial Performance is still the law. The Seller is entitled to payment even where the Tender of goods fails to conform exactly to the contract as long as it “Substantially” conforms.

Sec 2-612(1) “Installment Contract…(1) An Installment Contract is one which requires or authorizes the delivery of Goods in Separate Lots

to be Separately Accepted, even tho the Contract contains a clause “each delivery is a separate contract” or its equivalent.

Problem 52 Art Gallery ordered mthly shipments of statues from Ez Imports, agreeing to take 12-20 per month each over the next year. The 1st mth, the statues arrived shipped upside down; only one was broken and Ez Imports agreed to ship a replacement immediately. The next month the statues arrived shipped upside down again and ½ were broken. (1) Does 2-612 permit rejection for this reason.

(2) Given that Ez Imports replaces the broken statues within a week, but the next month,s shipment contained pictures instead of the ordered statues.

STOPPED CLASS HERE

- 31 -

Page 32: BSOL Fall 2010 Commercial Law Class Notes i and 2

B. Perfect Tender Rule – To prevail in a Single Delivery Sale, the Seller must make a Perfect Tender: One that complies with all of the terms of the contract, and then show that the Buyer refused to take the Goods.

Sec 2-601 Buyers Rights on Improper DeliveryS:t provisions of this Article on Breach in Installment contracts and UNLESS otherwise agreed under the sections on contractual limitations of remedy, if the Goods or the Tender of delivery fail in ANY respect to conform to the Contract, the Buyer may

a. Reject the whole; orb. Accept the Whole; orc. Accept ANY commercial unit or units and Reject the rest.

Problem 53Stella Speculator a wealthy investor signed a contract to purchase 5 new cars. All 5 were to be delivered on 10/01/01. When the cars arrived she test drove them and returned 2 of them. She rejected one b/c the cigarette lighter didn’t work; rejected the other b/c the trunk carpeting was ripped. Car Co offered to repair the 2 defects, but she refused. Car Co sued.

De Minimus rule will usually be applicable. Cigarette lighter is De Minimus

C. Cure – If Seller has NOT made a Perfect Tender and as a result the Buyer has rejected, Seller has a right in some circumstances to cure the defective Performance.

Sec 2-508 Cure by Seller of Improper Tender or Delivery; Replacement1. Where any Tender or Delivery by the Seller is rejected b/c Non-Conforming AND the time for

Performance has not yet expired, Seller may Seasonably Notify Buyer of his intention to Cure and may then within the contract time make a Conforming Delivery.

2. Where the Buyer rejects a Non-Conforming Tender which the Seller had reasonable grounds to believe would be acceptable with or without money allowance, the Seller may if he seasonably Notifies the Buyer have a further reasonable time to substitute a conforming tender.

Problem 54 The Ferdinands ordered a new car from Princ Motors on 8/1/01. Car was scheduled for delivery no later than 9/1/01. On 8/15 the car arrived. On their drive home, the engine blew up. The Ferdinands demanded their money back. Princ Motors made the following offers:(a) Replace the engine from another identical auto.(b) Refused to refund the money and replace the car with another one by 8/20/01.

See “Shaken Faith Doctrine”

ON EXAM “ Shaken Faith Doctrine ” – the purchase of a major Good is typically rationalized by the peace of mind that flows from its dependability. Once this faith is shaken, the Good becomes an instrument whose integrity is substantially impaired, per Zabriskie Chevrolet Inc v. Smith 240 A 2d 195

- 32 -

Page 33: BSOL Fall 2010 Commercial Law Class Notes i and 2

Wilson V Scampoli (p 214) ISSUE: Whether the Seller may conform his Tender by adjustment OR whether he must conform by substituting brand new merchandise.HELD: There are provisions of the UCC that indicate under certain circumstances repairs and adjustments are contemplated as remedies under Implied Warranties. The Seller should be able to Cure the defect under 2-508(2) in those cases where he can do so without subjecting the Buyer to any great Inconvenience, Risk or Loss.

D. Rejection and Acceptance When Seller makes Tender of Goods, Buyer can either Reject (2-602) or Accept (2-606-2-607); these are Mutually Exclusive actions. Failure to Reject acts in a technical Acceptance, since Rejection must come within a reasonable period of time after delivery of the Goods.

Sec 2-602 Manner and Effect of Rightful Rejection(1) Rejection of Goods MUST be within a Reasonable time AFTER their Delivery or Tender. It is

Ineffective UNLESS Buyer Notifies Seller.

(2) S:t 2-603 and 2-604 (a) After Rejection any exercise of ownership by Buyer w/r/t any commercial unit is wrongful as

against the Seller; and(b) If Buyer has BEFORE Rejection taken physical possession of Goods in which he does NOT

have a Security interest…he is under a Duty after Rejection to hold them with reasonable care at Seller’s disposition for a time sufficient to permit Seller to remove them; but

(c) Buyer has no further obligation with regard to Goods rightfully rejected.

(3) Seller’s rights w/r/t Goods wrongfully rejected are governed by 2-703.

KEY SECTIONSec 2-606 What constitutes Acceptance of Goods

(1) Acceptance of Goods occurs when the Buyer (a) AFTER a Reasonable opportunity to inspect the Goods signifies to Seller that the Goods are

Conforming or that he will take or retain them in spite of their Non-Conformity; or(b) Fails to make an effective Rejection, but such Acceptance does not occur until the Buyer has

reasonable opportunity to inspect them; or(c) Does any act inconsistent with Seller’s ownership; but if such act is wrongful as against the

Seller it is Acceptance ONLY if ratified by him.

(2) Acceptance of a part of any Commercial Unit is Acceptance of that Entire Unit.

Sec 2-607 Effect of Acceptance; Notice of Breach; Burden of establishing Breach after Acceptance(1) Buyer must pay at the contract rate for any Good Accepted

(2) Acceptance of Goods by Buyer precludes Rejection of Goods Accepted AND if made with knowledge of Non-Conformity CANNOT be revoked b/c of it UNLESS Acceptance was on reasonable assumption that the Non-Conformity would be cured…

(3) Where Tender has been Accepted(a) Buyer must within a Reasonable time after he discovers OR should have discovered any

Breach, Notify Seller of the Breach OR be barred from any remedy.

(4) The Burden is on Buyer to establish any Breach w/r/t Goods Accepted

- 33 -

Page 34: BSOL Fall 2010 Commercial Law Class Notes i and 2

Problem 55 M Seafoods in Iowa ordered 50 live Maine lobsters from M Exports “FOB Maine.” On 9/1/00 M Exports shipped to Des Moines; they failed to notify M Seafoods of the date of the flight until M Seafoods called to inquire as to their whereabouts 2 days later, where they had been sitting in Des Moines for 1 day. M Seafoods signed a receipt and picked them up. 20 Lobsters were dying (15 of these b/c of bad handling by M Exports before handing them over to be shipped, and an additional 5 due to damage in transit.) M Seafoods decided that it didn’t want any of them.(a) Is Seller’s failure to notify Buyer of the shipment grounds for Rejection?

ONLY if Material Loss or Delay resulted (Determination made based on “shelf life” of Goods: Goods have Shelf Life=Yes; If Widgets=No)

(b) May Buyer Reject b/c of the 20 defective Lobsters?Per 2-510(1). Since Lobsters damaged prior to shipment, even tho shipped “FOB Maine” Risk of Loss was still on the Seller, per 2-601. Therefore Buyer can Reject entire order.

(c) How quickly must Buyer act if it wished to Reject?Within a Reasonable time. Buyer must also take reasonable care of the Goods and make no further use of them

(d) Must Buyer reship the Good back to Seller if Seller offers to pay the freight?Since Buyer is a Merchant, it must take reasonable steps to deal with the rejected Goods; in doing so the Buyer is to be reimbursed for expenses.

(e) Can Buyer keep the 30 undamaged Lobsters for resale?Per 2-601, can Accept the 30 and Reject the 20.

(f) If Buyer rejects the Goods, must it give its reasons in Notice of rejection?It should

(g) If Buyer gives a valid Notice of Rejection within a reasonable time, what should it do with the shipment of Lobsters?

Buyer should take reasonable Care of the Rejected Lobsters; however, the Risk of Loss is still with the Seller.

Ramirez v Autosport ( p 219 )FACTS: Buyer traded in old camper for new camper. Seller to deliver by 8/3/78. When Buyer went to pick up it was scratched and utility hook-ups were not done. Buyer kept up with the progress of repairs yet it still was never ready. On 8/15/78 Seller transferred title to Buyer w/out Buyer’s knowledge. On 10/5/78 Buyer went to Seller rejecting the new camper and requesting their old camper back. Late in October Seller sold the old camper.

HELD: Seller is under a Duty to make a Perfect Tender and that a Buyer has the right to Reject Goods that do not conform to the contract. Nonetheless, that Rejection does not automatically terminate the contract; a Seller may still effect a Cure and preclude unfair Rejection and Cancellation by the Buyer. Rights of the Parties vary depending on whether Rejection occurs Before or After Acceptance of the Goods. Before Acceptance, the Buyer may Reject Goods for ANY Non-Comformity. Because of Seller’s right to Cure, however, Buyer’s Rejection does NOT necessarily discharge the contract.

The rights of the Parties also vary if Rejection occurs after the time set for performance. After expiration of that time the Seller has a further reasonable time to Cure if he believed reasonably the Goods would be Acceptable with or without a money allowance. After Acceptance, Buyer may Revoke ONLY if the Non-Conformity Substantially Impairs the Value of the Goods to him. Because a Buyer may Reject Goods with insubstantial defects, he may also cancel the the contract if those defects remain uncured. Otherwise a Seller’s failure to Cure minor defects would compel a Buyer to Accept imperfect Goods and collect for any loss caused by the Non-Conformity.

- 34 -

Page 35: BSOL Fall 2010 Commercial Law Class Notes i and 2

Problem 56 Simon ran a Dude Ranch and decided to erect a statue of a giant horse near the entrance to the ranch. The horse was specially mfgr’ed by Paris and arrived in 6 boxes. When it was put together Simon was displeased with the tail; it was different than what he had seen in the model. Simon removed the tail and substituted one of his own, returning the original tail to Paris with a letter of rejection. Simon painted the statue black and used it in his advertising. After 3 mths of display, and with no increase in business, Simon shipped it back to Paris with a letter of rejection stating that the problem with the tail made it unuseable.

(a) Did Simon make a Rejection or Acceptance?Buyer put the Horse up, painted it and put in a substituted tail, thereby treating it as his own. These acts equate to Acceptance.

(b) If tail did not conform to model, is that grounds for Rejection?Yes…but Buyer’s actions indicated that he Accepted the Good

(c) If Simon made a technical Acceptance, does that fact preclude a suit for Breach of Warranty?

No..Acceptance only precluded Rejection. Warranties are still required to be met.

Plateq Corp of North Haven (P) v. Machlett Labs Inc (p 229)FACTS: Seller of Specially Mfgr’ed Goods sued to recover purchase price from Commercial Buyer.

ISSUE: Whether Buyer accepted the Goods before it attempted to cancel the contract of sale for 2 lead covered steel tanks and their stands.

HELD: Once the tanks had been accepted, D could rightfully Revoke its Acceptance under 2-608 ONLY by showing substantial impairment of value to D. Acceptance of Goods occurs when Buyer (a) after a Reasonable Opportunity to inspect the Goods signifies to the Seller that he will take them in spite of their Non-Conformity; OR (b) fails to make an Effective Rejection. The Court chose to rely on 2-709(1)(b) which permits a price action for Contract Goods that cannot , after Reasonable Effort, be resold at a Reasonable price.

Specialty made goods, warranty, disclaimeri.e The “big peach” watertower in Chilton County

STOPPED CLASS HERE

- 35 -

Page 36: BSOL Fall 2010 Commercial Law Class Notes i and 2

E. Revocation of Acceptance – having made a technical Acceptance, a Buyer may still bring a Breach of Warranty provided that a proper 2-607(3)(a) Notice has been given.

Sec 2-607 Effect of Acceptance; Notice of Breach; Burden of Establishing Breach after Acceptance(3) Where a Tender has been accepted

(a) Buyer must within a Reasonable time AFTER he discovers or should have discovered any Breach, Notify Seller of Breach or be barred from any Remedy…

(4) Burden is on the Buyer to establish any Breach w/r/t Goods Accepted…

Sec 2-608 Revocation of Acceptance in Whole or in Part(1) Buyer may Revoke his Acceptance of a Lot or Commercial Unit whose Non-Conformity

Substantially Impairs it value to him if he has Accepted it(a) On the Reasonable assumption that its Non-Conformity would be Cured AND it has not been

Cured; or(b) Without discovery of such Non-Conformity if his Acceptance was reasonably Induced either

by the difficulty of discovery BEFORE Acceptance or by the Seller’s assurances.

(2) Revocation of Acceptance must take occur within a Reasonable time after the Buyer discovers or should have discovered the ground for it AND before any substantial change in condition of Goods which is not caused by their own defects It is not effective until the Buyer Notifies the Seller of it.

(3) A Buyer who so revokes has the same rights and Duties with regard to the Goods involved as if he had Rejected them.

Official Comment 2: Revocation of Acceptance is possible ONLY where the Non-Conformity impairs the value of the Goods to the Buyer. For this purpose, the test is NOT what the Seller had reason to know at the time of contracting, but whether the Non-Conformity is such as will in fact cause a Substantial impairment of value to the Buyer, tho the Seller had no advance knowledge as to the Buyer’s particular circumstances.

Sec 2-609 Right to Adequate Assurance of Performance(1) A contract for sale imposes an obligation on each Party that the other’s expectation of receiving

Due Performance will not be impaired. When Reasonable grounds for insecurity arise w/r/t the performance of either Party, the other may in writing demand adequate assurance of Due Performance and until he receives such assurance may…suspend any performance for which he has not already received the agreed return…

Rester v. Morrow (p 235)ISSUE: What are the rights of the Buyer and Duties of Seller of an auto that turned out to be a Lemon.What must the Buyer show before he may revoke his Acceptance and recover the purchase price; what proof must the Buyer show before he has created a question on his right to Revoke his acceptance.

HELD: Buyer’s entitlement to Revocation turns upon whether under the totality of the circumstances-temporarily and structurally the auto failed to be what the Seller was by private and public law obligated to provide Buyer and whether the aggregate Non-Conformity substantially impaired the value of the auto. Buyer of an auto is not bound to permit the Seller to tinker with the article indefinitely in the hope it may ultimately be made to comply with the Warranty. At some point in time, if major problems continue to plague the automobile, it must become obvious to all people that a particular vehicle simply cannot be repaired or parts replaced so that the same is made free of defect.

- 36 -

Page 37: BSOL Fall 2010 Commercial Law Class Notes i and 2

Problem 57 The day after Alice bought her new car, the right rear fender fell off.(a) May she use 2-608 or must she give dealer a right to Cure?

Depends whether the fender falling off is a Substantial ImpairmentPresume that dealer fixed the fender, but the first time it rained the paint washed off.(b) May she revoke now?

May be getting closer to Shaken Faith DoctrinePresume dealer fixed the paint, but 2 weeks later the engine stopped.(c) May she revoke now?

Getting easier to meet Substantial Impairment…

Problem 58 Same as last problem, but contract from dealer Explicitly limits the remedy for Breach to repair or replacement of defective parts. Dealer argues that defects have been repaired and replaced and remedy of Revocation is unacceptable to dealer and unavailable to Alice.

Problem 59

Problem 60

Problem 61 Arthur ordered computer # 740 from ION Corp; ION Corp sent him # 745, a newer model and better version of the computer he had ordered at same price. He sent them a letter of Acceptance along with a check. Once he began using it he was horrified to learn that it had an on/off switch under the front panel, as Arthur had witnessed his father’s loss of a finger due to a similar switch on another computer. Arthur sent a Notice of Revocation.

2-608 Official Comment 2 is clearly in Arthur’s favor since it adopts a Subjective test. Arthur clearly could have Rejected the Goods originally, but Revocation is harder to justify where Seller had no reason to know of the highly unusual problem.

Problem 62

F. Risk of Loss: Breach

Sec 2-510 Effect of Breach on Risk of Loss(1) Where a Tender or Delivery of Goods so fails to conform to the contract as to give a Right of

rejection, the Risk of their Loss remains on the Seller until Cure or Acceptance

(2) Where Buyer rightfully revokes Acceptance, he may to the extent of any deficiency in his effective insurance coverage, treat the Risk of Loss as having rested on the Seller from the beginning.

(3) Where the Buyer as to Conforming Goods already ID’ed to the contract for sale Repudiates or is otherwise in Breach before Risk of their Loss has passed to him, the Seller may to the extent of any deficiency in his effective insurance coverage treat the Risk of Loss as resting on the Buyer for a commercially reasonable time.

- 37 -

Page 38: BSOL Fall 2010 Commercial Law Class Notes i and 2

Jakowski (P) v. Carole Chevrolet, Inc. (D) (p 249) FACTS: P ordered a car that was supposed to have polymer undercoating. Seller delivered the auto on 5/19/80 w/out the undercoating. Seller contacted Buyer on 5/20/80 and told him to bring the car to the dealer to get the undercoating. On 5/22/80 Buyer brought the car back; it was stolen from the car lot that night.

ISSUE: Which Party bore the Risk of Loss.

HELD: Per 2-510(1), since the Goods failed to conform, the Risk of Loss never passed to Buyer. The rule under 2-510(1) is simple enough: Where Goods fail to conform to the contract of sale, the Risk of Loss remains on the Seller until the Buyer Accepts the Goods or until the Seller Cures the Defect.

Problem 63

G. Impossibility of Performance

Sec 2-613 Casualty to ID’ed GoodsWhere contract requires for its performance Goods ID’ed when contract is made and Goods suffer casualty without fault of either Party BEFORE Risk of Loss passes to Buyer…then

(a) if Loss is Total, contract is avoided(b) if Loss is Partial or Goods have so deteriorated as to no longer conform to the contract, Buyer

may nevertheless demand inspection and at his option either treat as avoided or accept the Goods with due allowance…for deterioration or deficiency in quality but without further rights against Seller

Sec 2-614 Substituted Performance(1) Where without fault of either Party the agreed loading…or carrier becomes unavailable or agreed

manner of delivery becomes…impractical, but a commercially reasonable substitute is available, such substitute…MUST be Tendered and Accepted.

(2) If the agreed means or manner of pymnt fails b/c domestic or governmental regulation, Seller may withhold or stop delivery UNLESS Buyer provides a means of payment…commercially equivalent

Sec 2-615 Excuse by Failure of Presupposed ConditionsExcept so far as Seller assuming a greater obligation…(a) Delay in delivery or non-delivery…by a Seller who complies with (b) and (c) is NOT a Breach of

his Duty under a contract IF performance has been made Impracticable by occurrence of a Contingency, the Non-Occurrence of which was a basic assumption on which the contract was made…

(b) Where the causes in (a) affect only a part of Seller’s capacity to perform, he MUST allocate production and deliveries among his customers…in any manner which is fair and reasonable

(c) Seller MUST Notify Buyer that there will be a delay or non-delivery, and when allocation is required under (b), the estimated quota available for Buyer

- 38 -

Page 39: BSOL Fall 2010 Commercial Law Class Notes i and 2

Official Comment 4 Increased Cost alone does NOT excuse performance UNLESS the rise on cost is due to some unforeseen Contingency which alters the essential nature of performance. Neither is a rise or collapse in the market itself justification…But a severe shortage of raw material or of supplies due…to war, embargo, local crop failure, unforeseen shutdown of major sources of supply…

Official Comment 5 Where a particular source of supply is exclusive…and fails thru Casualty, this section applies…There is no excuse…UNLESS Seller has employed all due measures to assure himself that his sources will not fail…

Sec 2-616 Procedure on Notice Claiming Excuse (1) Where Buyer receives Notification of material and indefinite delay or allocation…he may by

Written Notification to Seller as to any delivery concerned…(a) Terminate and discharge any unexecuted portion of the contract; OR(b) Modify the contract by agreeing to take available quota in substitution

(2) If after receipt of Notification from Seller, Buyer fails to modify the contract within a Reasonable time Not Exceeding 30 Days, the contract lapses w/r/t any deliveries affected

(3) Provisions of this section may NOT be modified except [where] Seller assumed Greater obligation…

Problem 64 VE ordered sundial from H Timepieces. All but 3 of the sundials were destroyed in an earthquake, and the 3 were slightly damaged. Upon learning of this VE demanded on looking at the 3 undestroyed sundials to select one for purchase, at a reduced amount.

2-615 is applicable section, since Goods were NOT ID’ed when contract was made (reqmnt under 2-613); Seller must apportion remaining sundials in a fair and reasonable manner.

Problem 65 Same as 64, but that H Timepiece put a “sold” tag on one of the sundials, which was the only one destroyed in the earthquake.

Since Goods were not req’d to be ID’ed when contract signed, 2-613 is N/A. He should get one of the remaining sundials.

ARABIAN SCORE v. LASMA (p 255)The horse died, pg. 258– “Doctrine of Commercial Frustration”

Impossibility of performance too, pg. 271

Problem 66 In mid-1960’s W Corp agreed to sell 80 million lbs of uranium over the next 20 years, with an avg sales price of $10/lb; W Corp only owned 15 million lbs. By mid 1970’s cost had risen to $40/lb and W Corp announced that it would not honor its contract, claiming 2-615 Commercial Impractability.

Cts have been reluctant to to let rising prices from Seller’s supplier be excuse for avoiding Fixed price contracts.

- 39 -

Page 40: BSOL Fall 2010 Commercial Law Class Notes i and 2

Louisiana Power & Light (P) v. Allegheny Ludlum Industries (D) (p 260)FACTS: breach of Contract case, where D was to supply stainless steel condenser tubing to P’s specs, on specified delivery dates. D sent P letter seeking additional compensation for performance under the contract; P refused to pay them. After P sent letter Notifying D that it considered the contract Repudiated by D, D wrote to P that D w/b will to sell at its cost; P refused and sought cover from another supplier.

ISSUE: Whether Commercial Impracticability applied in this situation.

HELD: There are 3 conditions that m/b met pursuant to 2-615 before performance under a contract can be excused b/c of Commercial Impracticability: (1) Contingency must occur; (2) Performance must be thereby made ‘Impracticable’; and (3) Non-Occurrence of the Contingency must have been a Basic Assumption on which the contract was made. Mere fact that performance wold have deprived D of its anticipated profits is NOT sufficient…The party seeking to excuse his performance must not only show that he can perform only at a loss, BUT also that the loss will be especially severe AND unreasonable.

CHAPTER 6 – REMEDIES (UCC Section 700+)

A. Special Remedies

1. Remedies on Insolvency – Reclamation

Sec 2-502 Buyer’s Rights to Goods on Seller’s Insolvency(1) S:t (2) AND even tho Goods have not been shipped, a Buyer who has paid a part or all of the price

of Goodsin which he has a special property…may on making a tender of any unpaid portion of the price, recover them from the Seller IF the Seller becomes Insolvent within 10 days AFTER receipt of the 1st installment on their price.

(2) If the ID creating his special property has been made by the Buyer, he acquires the right to recover the Goods ONLY if they conform to the contract for sale.

Sec 2-702 Seller’s Remedy on Discovery of Buyer’s Insolvency(1) Where Seller discovers the Buyer to be Insolvent, he may refuse delivery except for cash,

including payment for ALL Goods theretofore delivered under the contract, and stop delivery under this Article.

- 40 -

Page 41: BSOL Fall 2010 Commercial Law Class Notes i and 2

2. Liquidated Damages

Sec 2-718 Liquidation or Limitation of Damages(1) Damages for Breach by either Party may be Liquidated in the Agreement but ONLY at an amount

which is Reasonable in the light of the anticipated or actual harm caused by the Breach, the difficulties of proof of Loss and the inconvenience or nonfeasiblilty of otherwise obtaining an adequate remedy. A term fixing Unreasonably large Liquidated Damages is Void as a penalty.

(2) Where Seller justifiably withholds delivery of Goods b/c of Buyer’s Breach, Buyer is entitled to Restitution of any amount by which the sum of his payments exceeds:

a. amount to which Seller is entitled by virtue of terms [of liquidated damages]b. in Absence of such terms, (i) 20% of value of total performance for which Buyer is obligated

under the contract OR (ii) $500, whichever is Smaller.

(3) Buyer’s right to Restitution under (2) is s:t Offset to the extent Seller establishesa. a right to recover Damages under provisions of this article OTHER THAN (1), andb. amount of any benefits rec’d by the Buyer directly or indirectly by reason of the contract

(4) Where a Seller has rec’d payment in Goods their reasonable value or the proceeds of their resale shall be treated as payments for purposes of (2)…

3. The Breaching Buyer’s Restitution

Problem 67 Minerun Zoo contracted to buy an elephant by delivering a bear worth $300 and paying $100/mth for 20 mths at the end of which the zoo would receive the elephant. Minerun Zoo delivered the bear and made 15 mthly payments and then stopped for lack of funds. How much do they get back?

Since no Liquidated Damages stated, per 2-718(2)(b)(2) take LESSER of $500 or $460 (20% of total payments of $2,300) as Seller’s Liquidated Damages

STOPPED CLASS HERE

- 41 -

Page 42: BSOL Fall 2010 Commercial Law Class Notes i and 2

B. Seller’s Remedies

Sec 2-703 Seller’s Remedies in GeneralWhere Buyer WRONGFULLY Rejects or Revokes Acceptance of Goods OR Fails to make a payment due on or before delivery Or Repudiates w/r/t a part or the whole, then w/r/t any Goods directly affected and if the Breach is of the whole contract, then w/r/t the whole undelivered balance, the aggrieved Seller maya. Withhold delivery of such Goodsb. Stop delivery by any baileec. Proceed under 2-704 respecting Goods still unidentified to the contractd. Resell and recover Damages under 2-706e. Recover Damages for Non-Acceptance per 2-408 or in a proper case the price per 2-709f. Cancel

Sec 2-704 Seller’s Right to ID Goods to Contract notwithstanding Breach or to Salvage Unfinished Goods

(1) Aggrieved Seller may:(a) ID to the contract conforming Goods not already ID’ed if at the time he learned of the Breach

they are in his possession or control;(b) Treat as the subject of resale Goods which have demonstrably been intended for the particular

contract even tho these Goods are unfinished.

(2) Where Goods are Unfinished aggrieved Seller may in the exercise of Reasonable Commercial Judgment for the purpose of avoiding a loss...either Complete the manufacture and wholly ID the Goods to the contract OR cease manufacture and resell for scrap or salvage value or proceed in any reasonable manner.

1. Accepted Goods – In effect, Sec 2-709 is the equivalent of a Specific Performance remedy for the Seller

Sec 2-709 Action for Price(1) When the Buyer fails to pay the price as it becomes due, Seller may recover with any incidental

damages the price:(a) of Goods Accepted…and(b) of Goods ID’ed to the contract if the Seller is unable after reasonable effort to resell them at a

reasonable price…

(2) Where Seller sues for the price, he must hold for the Buyer any Goods which have been ID’ed to the contract and are still in his control, except that if resale becomes possible he may resell them at any time prior to collection of judgment.

Problem 68 Pg. 276 – BS Auto sold car to Dwayne on credit. One month after accepting his new sports car, he sent BS a notice of revocation because color of car clashed with his garage. He ceased making payments on car, parked car down the street and told BS Auto to come pick it up. 3 days after BS Auto received Notice the car disappeared and was never found.

Alleged Revocation is improper - not a good reason to reject, Dwayne also breached his 2-602 Duty to take reasonable care of the Goods, as he received the Risk of Loss upon receipt of the car. If Buyer is going to Revoke, he must take care of the Goods until the Risk of Loss passes back to the Seller.

- 42 -

Page 43: BSOL Fall 2010 Commercial Law Class Notes i and 2

2. Unaccepted Goods – where Buyer Repudiates before delivery or Rejects Goods

a. If Seller resells Goods see Sec 2-706b. If Seller does NOT resell the Goods see Sec 2-708

Sec 2-706 Seller’s Resale Including Contract for Resale(1) …Seller may Resell the Goods concerned or the undelivered balance thereof. Where resale is

made in Good Faith and in a Commercially Reasonable Manner, Seller may recover difference twix resale price and contract price together with any incidental damages allowed, but less expenses saved…

(2) Except under (3) or as otherwise agreed, resale may be at Public or Private sale…Resale must be reasonably ID’ed as referring to broken contract, but it is not necessary that Goods be in existence or that any be ID’ed to the contract before Breach.

(3) …[In a ]Private resale, Seller must give Buyer reasonable Notification of his intent to resell.

(4) Where the resale is at Public Sale:(a) only ID’ed Goods may be sold…(b) must be made at usual place or market for public sale…(c) …notification of sale must state place where Goods are located…(d) Seller may buy

(5) Purchaser who buys in Good Faith at a resale takes Goods free of any rights of the original Buyer…

Sec 2-708 Seller’s Damages for Non-Acceptance or Repudiation(1) S:t (2)…measure of damages for Non-Acceptance or Repudiation by Buyer is difference twix

Market price at time and place for Tender [KEY is the shipping terms to determine place of Tender] and Unpaid contract price plus incidental damages less costs saved.

(2) If measure of damages in (1) is inadequate to put Seller in as good a position as performance would have, then measure of damages is the profit which Seller w/h made from full performance by Buyer…

i.e For Special, One-of-a-kind or Unique Goods

ON EXAMProblem 69: Light Bulbs Inc (LBI), located in Austin contracted to sell 80 fixtures to Signs, Inc in San Antonio for shipment on 3/15/01. On 3/5/01 Signs, Inc called LBI and told them the deal was off, but LBI refused to agree to a cancellation. LBI went to warehouse and picked out 80 fixtures [since Seller weas able to just pick out from inventory, Goods are NOT Unique] and posted a sign next to its cash register stating that the 80 fixtures w/b sold to the person making the best offer. Carl saw the sign and made an offer of $1,000.

(a) Can LBI select Goods from warehouse AFTER Buyer Repudiates?Yes per 2-704.

(b) Was resale proper?No – Since Private resale, and no Notification as required by 2-706(3) was given

(c) (1) What are measure of damages under 2-708; (2) under 2-706?(1) Per 2-708, since Austin is place of Tender [shipping was FOB Austin] market price

on date of Tender of March 15 was $900; damages are $600 ($1500-$900)(2) Per 2-706 damages are $500 (diff twix resale price and contract price)

(d) Does LBI have choice twix 2-706 and 2-708 for calculating damages?

- 43 -

Page 44: BSOL Fall 2010 Commercial Law Class Notes i and 2

The Seller violating 2-706 Notification requirements is limited to a resale remedy.

Problem 70: Customer ordered a std above ground pool from FIS Pools, retailing for $2,000. Pool’s components are purchased wholesale by FIS Pools for $800; assembly costs by FIS Pools are $400. Customer has repudiated. Current market price is $2,000 and FIS is sure they can find another buyer.

FIS Pools is considered a “Lost Volume Seller” (see Teradyne, below, for discussion). The problem of a Seller with an unlimited supply of Goods is that since they can resell the Goods at regular price, they essentially lose the profit from this specific sale. The intent of 2-708(2) was to rescue this type of Seller and Cts have held this way for Seller.

KEY CASETeradyne Inc v. Teledyne Industries (p 278)HELD: It is universally agreed that in a case where after the Buyer’s default, a Seller resells the Goods, the proceeds of the resale are NOT to be credited to the Buyer if the Seller is a “Lost Volume Seller” – that is one who had there been no Breach by the Buyer, could and would have had the benefit of both the original contract and the resale contract. (i.e. a Seller who is able to satisfy all customers from stock.)

ON EXAMQuestion regarding Teadyne:1. College student who advertises his guitar for sale, contracts with caller to sell it, and then caller

backs out, is NOT a Lost Volume Seller, since he couldn’t satisfy ALL customers due to the fact that he had only 1 guitar to sell, not an unlimited supply.

Also: On Exam could get True/False question: The following are Lost Volume Sellers:Car Dealer (True)Pool Company (True)Limited Edition Sports Car Seller (False)

Problem 71: Computer Corp negotiated a contract whereby they would mfgr a special computer for a subway system in City of Plantation, Mississippi, price was $20,000 FOB Computer Corp’s plant in Atlanta. When computer was ½ complete, City had a change of administration and new leaders decided to dump the subway project and called to cancel the project. As scrap, the computer has a value of $5,000, it will cost $9,000 to complete and once finished it may be sold twix $15-$20K.

Should Computer Corp scrap it or complete it?Per 2-704(2) Seller must exercise Reasonable Commercial Judgment for the purpose of avoiding a loss; if it does so, it may recover the 2-706 damages.

If Seller completes and is unable to resell can it make City of Plantation pay?Per 2-709(1)(b) Seller can recover from City of Plantation.

- 44 -

Page 45: BSOL Fall 2010 Commercial Law Class Notes i and 2

C. Buyer’s Remedies

Sec 2-711 Buyer’s Remedies in General; Buyer’s Security Interest in Rejected Goods(1) Where Seller fails to make delivery OR Repudiates OR Buyer rightfully Rejects OR Justifiably

Revokes Acceptance Buyer may cancel and in addition to recovering the price as has been paid(a) “Cover” and have damages…as to all the Goods affected(b) Recover damages for Non-Delivery per 2-713

(2) Where Seller fails to deliver OR Repudiates, Buyer may also(a) If Goods are ID’ed, recover them per 2-502(b) In a proper case obtain Specific Performance or Replevy the Goods per 2-716

(3) On Rightful Rejection or Justifiable Revocation of Acceptance, Buyer has Security Interest in Goods in his possession for any payments made on their price…and may hold such Goods and resell them…

NOTE: Technically, a Buyer does NOT have to Cover; however, if a Buyer fails to Cover in an appropriate situation, Consequential damages that c/h/b avoided are denied

Sec 2-712 “Cover” Buyer’s Procurement of Substituted Goods(1) After Breach under 2-711, Buyer may “Cover” by making in Good Faith AND w/out

Unreasonable delay ANY Reasonable purchase or contract to purchase Goods in substitution for those due from Seller [Includes series of contracts as well as single contract or sale; Goods not identical but commercially useable as substitutes under the circumstances]

(2) Buyer may recover from Seller as damages the difference twix cost of Cover and contract price PLUS any Incidental or Consequential damages, BUT Less expenses saved in consequence of Seller’s Breach

(3) Failure of the Buyer to effect Cover under this section does NOT bar him from any other Remedy.

1. Accepted Goods

Sec 2-714 Buyer’s Damages for Breach in Regard to Accepted Goods(1) Where Buyer has Accepted Goods and given Notification per 2-607(3), he may Recover as

damages for any Non-Conformity of tender the loss resulting in the Ordinary Course of Events from the Seller’s Breach as determined in any manner which is reasonable.

(2) Measure of damages for Breach of Warranty is diff at Time and Place of Acceptance twix value of Goods Accepted and value they would have had if they had been warranted, UNLESS special circumstances show proximate damages of a different amount.

(3) …any Incidental and Consequential damages may be recovered.

- 45 -

Page 46: BSOL Fall 2010 Commercial Law Class Notes i and 2

Sec 2-715 Buyer’s Incidental and Consequential Damages(1) Incidental Damages include expenses reasonable incurred in Inspection, Receipt, Transportation

and care and Custody of Goods rightfully Rejected…expenses incurred with effectuating Cover…

(2) Consequential Damages include(a) any loss resulting from general or particular requirements and needs of which Seller at time of

contracting had reason to know and could not be prevented by Cover or otherwise; and(b) Injury to person or property proximately resulting from any Breach of Warranty.

NOTE: Important distinction twix Incidental and Consequential damages is that Courts have held where Contract contains a provision under which there is a disclaimer of only Consequential damages, this does NOT disclaim the Incidental damages.

Sec 2-717 Deduction of Damages from PriceThe Buyer on Notifying the Seller of his intention to do so may deduct ALL or ANY part of the damages resulting from any Breach of the contract from any part of the price still due under the same contract.

Problem 73: World-famous pianist Bart made $50K/yr giving concerts. He decided to experiement with new sounds and ordered a new electronic piano. Purchase was negotiated for price or $3,000 and there was no written contract [1st thought is that Statute of Frauds req’d a written contract since Goods> $500]. After 3 mths he noticed that he was losing his hearing; cause was a high pitch from the electronic piano. He destroyed the electric piano and brought suit under Breach of Warranty asking for the following. $3,000, the cost of the piano; $2,000 doctor’s fees; $500 fees paid to experts to examine piano and determine whether it was the cause of his problem; $750,000, 15 yrs worth of income at $50K/yr; $1M for loss of hearing; and $5 for ax handle busted.

What Warranty, if any, was Breached?Merchantability, altho issue will be whether the piano was fit for its ordinary purpose. If so, defense would be that this type of injury was Unforseen Freakish accident.

Which damages are recoverable under 2-714 ?Only the cost of the electric piano

Which damages are recoverable under 2-715(1) ?Most likely the $500 fees paid to experts to examine piano and determine

Which damages are recoverable under 2-715(2) ?Most likely $2,000 doctor’s fees; also potential for either lost income or loss of hearing (after appropriate discounting for present value)

2. Unaccepted Goods

Problem 76: Mr T ordered a 1999 Blocklong mobile home fro $8,000 from HW Sales Co, delivery to be on 5/20/01. Mr T planned on spending addt’l $500 to build foundation that the Blocklong had to have for maximum utility. Due to widespread industry strikes, prices of trailers rose dramatically, and on 5/10/01 HW Sales Co notified Mr T that the deal was off. Mr T shopped around and on 9/5/01 bought a 1999 Bohemoth model for $15,000 from another dealer. The Bohemoth was larger than the Blocklong and did not require a foundation. Mr T brought suit and HW Sales Corp defended by stating that the Bohemoth was selling for $10K up to 9/5/01 when the price rose to $15K; and the Bohemoth always sells for $2K more due to its larger size and better features.

Fact that Behemoth does not require a foundation is an Expense Saved; additionally the

- 46 -

Page 47: BSOL Fall 2010 Commercial Law Class Notes i and 2

$2,000 price differential could be argued by HW Sales Co as additional value that inures to Mr T’s benefit and would reduce damages; also the delay in Coverage could be argued as a mitigating factor in the $5k price increase. Mr T could wind up with no damages.

STOPPED CLASS HERE

- 47 -

Page 48: BSOL Fall 2010 Commercial Law Class Notes i and 2

Tongish v. Thomas HELD: The “Market Damages remedy” should be followed as the preferred measure of damages…it permits the Parties to measure the Expectancy of what might have happened if the Seller does not perform the contract. The Buyer has an option at the beginning of the contract to take actions to protect against an uncertain future. The Parties both know that the option is an election that can be exercised by the Buyer to protect against future losses.

D. Anticipatory Repudiation – a Repudiation must be a “definite refusal to perform”;Mere Equivocation is NOT enough.

Sec 2-610 Anticipatory RepudiationWhen either Party Repudiates the contract w/r/t a performance not yet due, the loss of which will

substantially impair the value of the contract to the other, the aggrieved Party may(a) for a Commercially Reasonable time. await performance by the repudiating Party; or(b) resort to any remedy for Breach, even tho he has Notified the Repudiating Party that he would

await the latter’s performance and has urged retraction; and(c) in either case suspend his own performance or proceed in accordance with the provisions of

this article on Seller’s right to ID Goods to the contract notwithstanding Breach or to salvage unfinished Goods.

Official Comment 1…Anticipatory Repudiation centers upon an overt communication of Intention or an action which renders performance impossible, or demonstrates a clear determination NOT to continue with performance...[if aggrieved party] awaits performance beyond a Commercially Reasonable time, he CANNOT recover resulting damages which he could have avoided.

Official Comment 2 Repudiation can result from action which reasonably indicates a rejection of the continuing obligation…and automatically results…when a Party fails to provide adequate assurances of due future performance w/in 30 days after a justifiable demand therefore has been made.

Sec 2-611 Retraction of Anticipatory Repudiation(1) Until the Repudiating Party’s next performance is due, he can Retract his Repudiation

UNLESS the aggrieved Party has since the Repudiation Cancelled OR Materially Changed his Position [i.e “Covered”] OR otherwise indicated that he considers the Repudiation final.

(2) Retraction may be made by any method which Clearly indicates tot he aggrieved Party that the Repudiating Party intends to perform, but MUST include any assurance justifiably demanded…

(3) Retraction reinstates the Repudiating Party’s rights under the contract with due excuse and allowance to the aggrieved Party for any delay occasioned by the Repudiation.

- 48 -

Page 49: BSOL Fall 2010 Commercial Law Class Notes i and 2

Sec 2-713 Buyer’s Damages for Non-Delivery or Repudiation(1) S:t proof of Market Price under 2-723, Measure of damages for Non-delivery or Repudiation by

Seller is diff twix Market Price at time Buyer learned of Breach and Contract price PLUS any Incidental and Consequential damages LESS expenses saved as consequence of Seller’s Breach

(2) Market Price determined at time of place for tender, or, in cases of Rejection after arrival or revocation of Acceptance, as of the place of arrival.

NOTE: Cts have interpreted the “at time Buyer learned of Breach” language as meant to apply ONLY in situation in which the Buyer first discovered the Breach after the due date set for performance.

Sec 2-723 Proof of Market Price: Time and Place(1) …shall be determined according to the price of such Goods prevailing at the time the aggrieved

Party learned of the Repudiation.

(2) If evidence if a price prevailing at the ties or places…is NOT readily available, the price prevailing w/in any reasonable time before or after …or at any other place…may be used, making proper allowance for the cost of transporting the Goods…

Problem 79: On 4/08/01, the Army contracted with Cattle Co for 1000 lbs of beef for delivery 10/08/01 at a price of $5,000. Shortly thereafter price of beef rose sharply and Cattle Co Repudiated on 7/8/01 when beef price was $6,000. Army looked for Coverage and found that on 7/15/01 it w/h cost them $7,000. Instead Army sent Cattle Co a note that they did not recognize the Repudiation and expected performance on 10/08/01. By 10/08/01 price had risen to $8,000; upon nonperformance by Cattle Co Army decided to serve beans.

Determination will be when was a Commercially Reasonable Time for Coverage after Buyer learned of Breach.

E. Statute of Limitations

Sec 2-725 Statute of Limitations in Contracts for Sale(1) An action for Breach…must be commenced within 4 yrs AFTER Cause of Action has accrued. By

the original agreement the Parties may reduce the period of limitation to NOT LESS THAN 1 yr, but may NOT extend it.

(2) Cause of Action accrues when the Breach occurs, regardless of the aggrieved Party’s lack of knowledge of the Breach. A Breach of Warranty occurs when Tender of delivery s made, EXCEPT that where a Warranty Explicitly extends to Future performance of the Goods and Discovery of the Breach must await the time of such performance the Cause of Action Accrues when the Breach is OR should have been discovered.

NOTE: Where a Warranty given with the sale of a Good gives a time period, the 4 yr clock starts ticking at the moment within that period when the breach s/h/b discovered.

Nationwide Ins Co v. Gen’l Motors Corp/Chevrolet Motor Division (p 303)HELD: The phrase Explicitly extends to Future performance can be interpreted to include a promise that by its terms comes into play upon, or contingent upon, the future performance of the Goods.

- 49 -

Page 50: BSOL Fall 2010 Commercial Law Class Notes i and 2

Problem 81: Jane bought a new car on 4/1/02; written Warranty that came with the car read : “The mfgr will replace any part found to be defective within the first 5 years.” 3 yrs and 358 days after Jane purchased the car the steering wheel came off in her hands. She had the car towed to the dealer that day. Dealer kept the car for 3 mths, promising each week that it would be repaired. After picking it up form the dealer, while driving it home, the wheel came off again and Jane was killed in an accident.(a) When did the action accrue w/r/t the steering wheel?(b) Was the statute tolled during the 3 mths that the car was in the shop?(c) If mfgr of car sold it to dealer and dealer in turn sold to consumer, would the 4 yr period on

mfgr’s Implied Warranty start running on the date of delivery to the dealer or on the date of the ultimate sale to consumer? What if Warranty was Express Warranty?

This is the “Shelf Life” problem. Cts have favored that the S of L on Mfgr’s Warranty runs out 4 yrs after sale to retailer.

(d) Should Ct draw distinction as to when Cause of Action accrues based on whether the injury is to Property or Person?

Central Washington Refridge v. Barbee (p 317)HELD

- 50 -

Page 51: BSOL Fall 2010 Commercial Law Class Notes i and 2

Summary of Semester:

Thing is a GOOD and is covered under Article II of UCC

MERCHANT – held to objective standard that includes observance of fair dealings in that trade. UCC Article II – regularly deals, special knowledge of goods or has employee w/ knowledge

Offer and acceptance

Typical Question Analysis: I. Contract – is there one present?II. In agreement on presence of contract?III. Custom and UsageIV. UCC rulesV. What performance is required?VI. Can it be cured?

Course of Performance

Goods – Tangible chattels

2-316 – Disclaimers

2-313 – Express warranties

2-201 – Statute of Fraud – is there a contract

2-201 3(a) – Specially made goods and substantial performance

2-201 3(c) – Accept or receipt of payment

Goods – “all tangible chattels” “moveable”Except 2-201 subsection 3A “specially made goods”Except 2-201 subsection 3C “offer acceptance”

- 51 -

Page 52: BSOL Fall 2010 Commercial Law Class Notes i and 2

Summary of 1st 104 Pages:QUESTIONS

T o F Goods include all moveable items at time of contract formation

A contract for crops is contract for goods regardless the crops are to be harvested by seller or buyer

A writing that doesn’t contain a hand written signature doesn’t satisfy Statue of Frauds. Sec 2-201

Evidence or course of dealing or usage may not be offered to explain memo that was supposed to be final expression

Unless otherwise agreed, seller determines the place of delivery of goods

Good Faith in case of merchants means only honesty in fact

Facts: Seller agrees to sell 10,000 – contract in writing shipping arrangement and truck wouldn’t work so sent by rail. Goods weren’t delivered to address and buyer canceled.

Question 1: Did buyer have right to end contract? Yes, not delivered to correct address. Can’t change price due to shipping. Make new arrangements if foresee contract breached.

Question 2: Can seller prevent buyer from canceling? Date for performance important. No, if delivered at specific location and correct date, time.

Acts of God can limit UCC expectations. Hurricane limiting delivery

Deliver to wrong address – no damage to goods – usually ok. If you have to have goods and buy from competitor for more – seller has to recompensate for buyer having to pay more.

Substituted Goods – buyers choice to accept or claim breach of contractStart of performance – assent to contract

DIAMOND FRUIT GROWERS V KRACK CORPORATIONPrevious course of dealings for 10 years. Pg 58 “sound policy…”, “last shot” rule – who sends has last say – not fair.

Pg. 68 – can change if you want – don’t sign

Statute of Frauds – over $500

Sec 2-207 pg. 73

Pg. 74 keep terms consistant – if terms not agreeable – “Gap Filler” provisions of Article II

Warranties of Title

Pg. 78 Upstream Remedy – go back to fast Eddie or Sealed Lips. Title has to be legitimate.

Problem 19 p 78– KNOW

Warranties of Quality: Express – does something affirmative to sell verbal sales pitch, advertisement, written contract, “puffing” or express warranty Sec 2-313

- 52 -

Page 53: BSOL Fall 2010 Commercial Law Class Notes i and 2

Problem 20

a) how much knowledge buyer has of good has a lot to do with “puffing” or not, did buyer RELY on statement by seller, if you are an expert in cars or whatever – courts won’t be as lenient

b) Express; if in writing – JUDGE *

if oral – JURY *

c) no – puffing

Problem 22 (QUESTION ON EXAM) Warranty came after car so – not bound by limited warranty, equal or superior knowledge – warranty

Problem 23 he never saw ad so – no express warranty, not a basis of bargain

Implied Warranties – implied as a matter of law

Merchantability: Must be saleable and conform to normal expectations of both parties, Sec 2-314(2) food and drink are included here – clam chowder – shell included (expected); wine and wine glass have to be merchantable, wrapper container, too

SHAFFER V VICTORIA STATION

Wine glass broke upon first sip – (the wine glass has to be merchantable too)

Sec 2-314(2) fungibles – grains, oats, corn; product has to be used for its intended use (chainsaw – wood, not for ice sculpture)

Sec 2-315 Fitness for a particular purpose – (p. 92) if sellers says you can use it for additional purposed, stores like Home Depot have a certain liability for selling products like deck kits

Problem 26 –

Jones – merchant Sec 2-314 – not breached, it worked. Sec 2-315 – Wren decided he wanted this one but Jones knew room well – Wren relied

on Jones

Problem 27

Smelly paint would breach Sec 2-314(2)(c) non-matching color would breach Sec 2-315

Problem 28

Natural substance – olive pit in martini; Just break a tooth because crunchy food – no cause for recovery Foreign object –box staple – cause for recovery

Pg. 98 Question #3. Stricter standard for canned goods

Statute of Frauds – over $500 and both have to be merchants

- 53 -

Page 54: BSOL Fall 2010 Commercial Law Class Notes i and 2

Sec 2-105 both merchants

Sec 2-102 make an oral agreement between two merchants and A sends B a confirmatory written document and B doesn’t do anything within 10 days – contract stays and is valid

Goods are EVERYTHING that are moveable at the time of sale. Stocks – not goods

Warranty suit – P has burden of proving: Creation of Warranty Its breach

Proximate causeFacts and extent of injury

FLIPPO V MODE O’DAY (pg. 99)Spider wasn’t part of pants (pg. 102) – therefore no cause for action under UCC Article 2

NOTE: Proper way to avoid liability for an express warranty is to not make it in the first place (pg. 104)

- 54 -